TEST PAPER 6 Flashcards

1
Q

1.A Computed tomography (CT) chest is done in a 6-month-old girl with a history ofpremature birth, chronic lung disease and MRSA pneumonia. It shows a large gas-containing,thin-walled cavity in the right lung, consistent with a pneumatocele. All the following aretrue regarding a pneumatocoele, except:
A. They are gas-filled, thin-walled spaces surrounded by lung.
B. They can be associated with lung contusion.
C. Most pneumatocoeles resolve spontaneously.
D. Pneumatocoeles do not cause mediastinal shift.
E. They can have fluid levels.

A
  1. D. Pneumatocoeles do not cause mediastinal shift.
    Pneumatocoeles typically are post-infectious or post-traumatic, discrete, thin-walled,gas-containing collections within the lung parenchyma. They also may result from positive pressure ventilation-related barotrauma and ingestion of caustic material (e.g. hydrocarbons). Post-infectious pneumatocoeles most frequently complicate staphylococcal pneumonia and occur in infants. They typically appear within 1 week of onset of infection and most spontaneously disappear within weeks to months after the infection has resolved. Rarely, persistent pneumatocoeles may require percutaneous catheter drainage or surgical management. Post-traumatic pneumatocoeles result from blunt trauma. Such pneumatocoeles are typically observed within hours of the trauma and spontaneously resolve within 3 weeks. They generally spare the lung apices.
    At CT, pneumatocoeles appear as well-defined parenchymal cystic structures, with a thin wall. They may be entirely filled with gas, or an air-fluid level may be seen. Contralateral mediastinal shift may be seen with large pneumatocoeles. They are associated with contusion in blunt trauma and can rupture to cause pneumothorax.
How well did you know this?
1
Not at all
2
3
4
5
Perfectly
2
Q

2.A 58-year-old patient is reported to have a carcinoid tumour of the gastrointestinal (GI)tract on an abdominal computed tomography scan. What is the most common primary site ofGI carcinoids?
A. Stomach
B. Colon
C. Rectum
D. Small bowel
E. Appendix

A

2.E. Appendix
Carcinoid tumours are the most common primary tumour of the small bowel and appendix.Gastrointestinal carcinoids account for about 85% of all carcinoid tumours, with the remaining 15% occurring in the lungs and bronchi. These tumours arise from the enterochromaffin cells of Kultchitsky; these express serotonin and other histamine-like substances. The appendix is the most common site of carcinoids, accounting for 30%-45%, the small bowel for 25%-35%, the rectum 10%—15%, the colon 5% and the stomach <3%. Most tumours are clinically silent but may cause pain, obstruction, weight loss and, rarely, bowel perforation. In rare cases (7% of small bowel carcinoids), the hormonal load from the tumour may overwhelm the liver’s capacity to metabolise serotonin, causing a carcinoid syndrome recurrent diarrhoea, right-sided endocardial fibroelastosis, wheezing/bronchospasm and flushing of the face and neck.

How well did you know this?
1
Not at all
2
3
4
5
Perfectly
3
Q

3.A 53-year-old man with history of haematuria which shows a gel-like polypoid filling defecton cystoscopy is sent for an MRI. The MRI shows a low Tl signal, heterogeneous T2 signal(central high and peripheral low) lesion. On post-contrast Tl W FS images, the peripheral portionenhances more than the central potion, resembling a ring-like pattern. What is the diagnosis?
A. Endometriosis
B. Inflammatory pseudotumour
C. Malakoplakia
D. Cystitis glandularis
E. Eosinophilic cystitis

A

3.B. Inflammatory pseudotumour
Inflammatory pseudotumour is an interesting entity that has been reported in every organ of thebody. At imaging, it usually appears as a solitary exophytic or polypoid bladder mass, which may be ulcerated. On T2-weighted MRI, it is heterogeneous, with a central hyperintense component surrounded by a low-signal-intensity periphery; on post-contrast images the periphery enhances, whereas the central region enhances poorly. The central region consists of necrotic tissue, and the periphery comprises fascicles of spindle cells in oedematous stroma with myxoid components, vessels and inflammatory cells (hence the name pseudosarcomatous fibromyxoid tumour). This structure may produce the pattern of ring-like enhancement observed on CT and MR images suggestive of the diagnosis, but histologic confirmation is essential. In young adults, the presence of luminal clot surrounding an enhancing bladder mass may also suggest this diagnosis.
MRI shows single or multiple masses. On T2-weighted images, cystitis glandularis shows low signal intensity with a central branching high-signal pattern. The hyperintense area shows the most contrast enhancement and corresponds to the vascular stalk. Eosinophilic cystitis nodules are hyperintense to muscle on T1, isointense on T2-weighted images, and enhanced after intravenous contrast administration.

How well did you know this?
1
Not at all
2
3
4
5
Perfectly
4
Q

4.A 33 year-old man presented with fever and sudden-onset back pain. Inflammatorymarkers were slightly raised. MRI spine showed high signal in the L3/4 disc space on T2Wand STIR images, with enhancement of the disc extending to the adjacent end plates onpost contrast sequences. What is the diagnosis?
A. Metastasis
B. Prolapsed intervertebral disc
C. Sequestrated disc
D. Discitis
E. Epidural abscess

A
  1. D. Discitis
    Pyogenic spondylitis most commonly involves the lumbar spine and one spinal segment, which consists of two vertebral bodies and the intervening disk. It typically displays low signal intensity on T1-weighted images, with a loss of definition of the vertebral end plate and of the adjacent vertebral bodies and high signal intensity on T2-weighted images. In the involved disk space, fluid- like signal intensity is seen on both Tl and T2 images. Following the intravenous administration of gadolinium-based contrast material, disk enhancement patterns from homogeneous to patchy nonconfluent to peripheral enhancement may be seen. Infected bone marrow also enhances diffusely after contrast material is administered; contrast-enhanced fat-suppressed MR images are especially useful in demonstrating this marrow abnormality. MR imaging provides better definition of epidural extension of the inflammatory process and compression of the spinal cord and dural sac than other imaging modalities do. Paravertebral and epidural extension may appear in the form of either a phlegmon or an abscess with mixed signal intensity on both T1-weighted and T2-weighted images.
How well did you know this?
1
Not at all
2
3
4
5
Perfectly
5
Q
  1. A 65-year-old woman with progressive increase in knee pain and limited mobility is referred by her GP to have a plain X-ray of the knee. Plain films show bilateral chondrocalcinosis along with some other arthritic features. Which one of the following conditions is the most common cause of chondrocalcinosis?
    A. Calcium pyrophosphate dihydrate crystal deposition disease
    B. Hydroxyapatite crystal deposition disease
    C. Primary synovial osteochondromatosis
    D. Intraarticular synovial cell sarcoma
    E. Chronic renal failure
A
  1. A. Calcium pyrophosphate dihydrate crystal deposition disease
    Pseudogout [calcium pyrophosphate dihydrate (CPPD) deposition disease] is a syndrome caused by the deposition of CPPD crystals in and about the joints of middle-aged or older adults. Three discrete manifestations of this deposition of CPPD crystals are recognised:
    (1) chondrocalcinosis, (2) typical arthropathy and (3) the clinical presentation of pain. Any combination of these features may suffice to suggest the diagnosis of pseudogout.
    Other causes of chondrocalcinosis include the following:
    Hyperparathyroidism Gout
    Wilson disease
    Haemochromatosis
    Ochronosis
    Trauma
    Osteoarthritis
    Hypothyroidism
    Hypomagnesaemia
    Acromegaly
    Oxalosis and hydroxyapatite deposition disease (HADD)
How well did you know this?
1
Not at all
2
3
4
5
Perfectly
6
Q
  1. A 14-year-old boy with family history of pulmonary chondroma in his elder brother is investigated with a CT of the chest and abdomen. The CT shows a 3 x 3 cm calcified perihilar lung mass and a large mixed density mass in the left upper quadrant, anterior to the spleen inseparable from the stomach. What other finding(s) would you expect on the CT?
    A. Bilateral renal carcinoma
    B. Hepatoblastoma
    C. Multiple cysts in the lung, kidney and pancreas
    D. Wilms tumour on the right
    E. Multiple extra-adrenal neuroblastomas
A
  1. E. Multiple extra-adrenal neuroblastomas
    The question describes the Carney triad: pulmonary chondroma, gastric GIST and multiple extra-adrenal neuroblastoma.
    Cancer predisposition syndrome (CPS) is the term that is generally reserved to describe familial cancers in which a clear mode of inheritance can be established. Individuals may present with one or more key physical features or congenital anomalies (e.g. hemihypertrophy). Patients may have specific tumours that are known to be highly associated with a CPS (e.g. haemangioblastomas in VHL (Von Hippel-Lindau syndrome) disease).
    Some physical features suggesting CPS are cafe-au-lait spots (NF1, NF2, Bloom s), angiofibromas (tuberous sclerosis, TS), pits in palms and soles (Gorlin), macrocephaly (Sotos, Cowden, Gorlin), macroglosia [BWS ( (Beckwith-Wiedemann syndrome)], hyperpigmentation (NF1, Fanconi anaemia, Blooms), spotty skin pigmentation (Carney complex), hemihypertrophy (NF1, BWS, Klippel Trenaunay syndrome), thumb malformation (Fanconi anaemia), aniridia [WAGR (Wilms tumor- anirida syndrome with genitourinary anomalies)] and so on.
    Tumours associated with CPS include Wilms tumour (WAGR, BWS and several others), haemangioblastoma (VHL), dear cell renal carcinoma (VHL, TS), pheochromocytoma (VHL, MEN 2B, NF1), hepatoblastoma (BWS, Familial adenomatous polyposis, FAP), adrenocortical and breast carcinoma (Li-Fraumeni syndrome, LFS), optic glioma and neurofibrosarcoma (NF1), retinoblastoma (familial retinoblastoma), gastric cancer and GIST (FAP, NF1, Carney triad, HNPCC (hereditary non-polyposis colorectal cancer] or lynch syndrome, LFS, MEN1), neuroblastoma (NFl, BWS), rhabdomyosarcoma (LFS, NF1, BWS, hereditary retinoblastoma) and so on.
    Screening tests and pathways have been established for several of these CPSs. These include US abdomen for BWS, LFS, FAP and VHL; prophylactic thyroidectomy for MEN 2; and so on.
How well did you know this?
1
Not at all
2
3
4
5
Perfectly
7
Q
  1. Transverse US image of a foetal thorax with a four-chamber view of the heart demonstrates homogeneous intermediate echogenicity of the right lung. The heart is mildly rotated to the right and there are cystic areas in the left side of the thorax suggesting a diaphragmatic hernia. All of the following are true regarding investigation of congenital lung anomalies, except;
    A. Meconium shows high T1 and low T2 signal on MRI.
    B. Normal lung reduces in T2 intensity as it matures.
    C. Secondary pulmonary hypoplasia is more common than primary’.
    D. Interventricular septum determines the cardiac axis.
    E. Echogenicity of lung advances as gestation advances
A
  1. B. Normal lung reduces T2 intensity as it matures.
    At US, the foetal lungs normally appear homogeneous and are slightly more echogenic than the liver. The echogenicity of the lung increases as gestation advances. The presence of cysts or focal increased echogenicity of the lung parenchyma indicates a mass. The axis of the heart is determined relative to the interventricular septum.
    At MR imaging, the trachea, bronchi and lungs demonstrate high T2 signal intensity relative to the chest wall muscles, because they contain a significant amount of fluid. As the lungs mature, there is increasing production of alveolar fluid, thereby increasing the T2 signal intensity of lung relative to the liver.
    Pulmonary hypoplasia can be primary or secondary. Primary pulmonary hypoplasia is less common. The most common intrathoracic cause of secondary pulmonary hypoplasia is congenital diaphragmatic hernia. The herniated liver can be confused with a mass originating in the lung. Colour Doppler imaging may be helpful in identifying the portal and hepatic veins.
    Meconium-filled large bowel is hyperintense on T1-weighted images and hypointense on T2 weighted images; therefore, intrathoracic herniation of the large bowel can easily be detected at MR imaging.
    The most common extrathoracic cause of pulmonary hypoplasia is severe oligohydramnios, secondary to cither foetal urogenital anomaly or premature rupture of membranes.
How well did you know this?
1
Not at all
2
3
4
5
Perfectly
8
Q
  1. A 42-year-old man who sustained a comminuted acetabular fracture underwent a CT of his pelvis for further characterisation and treatment planning. The CT report described it as an anterior column acetabular fracture. Which one of the following anatomic structures must be disrupted on the CT?
    A. Ilioischial line
    B. Iliopectineal line
    C. Sacroiliac joint
    D. Anterior wall
    E. Posterior wall
A
  1. B. Iliopectineal line
    On radiographs, the iliopectineal (or iliopubic) line represents the border of the anterior column, and the ilioischial line represents the posterior column.
    Fracture involvement of the anterior and posterior columns is characterised by disruption of the iliopectineal line and ilioischial line, respectively. However, disruption of these lines may also be seen with other fracture patterns, such as a transverse fracture. Obturator ring and iliac wing involvement must also be present for classification as a both-column acetabular fracture.
How well did you know this?
1
Not at all
2
3
4
5
Perfectly
9
Q
  1. A 29 year old woman has come to the A&E department with 3 months’ history of shortness of breath after her second miscarriage. She had an episode of pulmonary embolism during her first pregnancy and epilepsy in her teens. Chest radiograph done in the A&E department shows progressive enlargement of cardiac silhouette and left-sided pleural effusion. What is the likely diagnosis?
    A. Systemic lupus erythematosus (SLE)
    B. Rheumatoid arthritis
    C. Wegner’s disease
    D. Polyarteritis nodosa (PAN)
    E. Homocystinuria
A
  1. A. Systemic lupus erythematosus (SLE)
    Systemic Lupus Erythematosus (SLE) patients with antiphospholipid antibody (aPL-ab) syndrome present with arterial and veno-occlusive disease, thrombocytopenia and recurrent vascular thromboses and miscarriages. Patients with aPL-ab syndrome can present with recurrent strokes, Budd-Chiari syndrome, dural venous sinus thrombosis, ischaemic bowel and recurrent pulmonary embolism. Exudative pericardial effusions and pericarditis are common. Pleural effusions are the most common manifestation of SLE in the respiratory system and are bilateral in approximately 50% of patients.
    Epileptic seizures are seen in 11% patients with SLE, with association of aPL-ab syndrome and stroke.
How well did you know this?
1
Not at all
2
3
4
5
Perfectly
10
Q
  1. Abdominal computed tomography (CT) in a 57-year-old patient with non-specific abdominal pain demonstrates an elongated cystic mass in the expected region of the appendix. The lesion appears to be invaginating into the caecum and demonstrates curvilinear calcification in its wall. What is the most likely diagnosis?
    A. Lipomatosis of ileocaecal valve
    B. Carcinoid tumour of the appendix
    C. Mucocoele of the appendix
    D. Epiploic appendagitis
    E. Myxoglobulosis
A
  1. C. Mucocoele of the appendix
    Mucocoele of the appendix is an umbrella term used for the appearance of a cystic mass within the appendix that has varying pathological cause. Mucocoeles may be secondary to mucosal hyperplasia (25%), mucinous cystadenoma (63%) and mucinous cystadenocarcinoma (12%). The HU (Hounsfield Unit) value of the cystic lesion is variable from water density to soft tissue density depending on the volume of mucin present within it. A CT scan is good at demonstrating the curvilinear or punctate rim-like calcification that is present in approximately 50% of cases. Ultrasound may demonstrate a right lower quadrant lesion with either cystic or mixed internal echogenicity depending on the amount of mucin. Rupture of mucocoeles may lead to pseudomyxoma peritonei with characteristic findings of multiple thin-walled cystic masses of varying sizes in the abdomen, scalloping of the liver and splenic margins and a gelatinous ascites. Myxoglobulosis is a rare variant of mucocoele with characteristic small, rounded calcific spherules.
How well did you know this?
1
Not at all
2
3
4
5
Perfectly
11
Q
  1. Tuberculous spondylitis is diagnosed in a 44-year-old woman with progressive neurological deficit with severe discovertebral destruction and compression of the spinal cord at the Tll-12 level on sagittal T2W & STIR MR images. Post-contrast images show a rim-enhancing anterior abscess that does not encase the intercostal arteries. All of the following features are more likely to represent tuberculosis spondylitis compared to pyogenic spondylitis, except:
    A. Subligamentous spread
    B. Three or more vertebral level involvement
    C. Skip lesions
    D. Homogenous enhancement of the disc
    E. Paraspinal calcification
A
  1. D. Homogenous enhancement of the disc
    Spinal tuberculosis most commonly involves the thoracic spine and less often the lumbar spine. It is often difficult to differentiate between tuberculous and pyogenic spondylitis, both clinically and on images. MR imaging is very helpful for differentiating between tuberculous spondylitis and pyogenic spondylitis. A well-defined paraspinal mass with abnormal signal intensity; a thin, smooth abscess wall; subligamentous spread to three or more vertebral levels; and multiple vertebral or entire-body involvement are findings more suggestive of tuberculous spondylitis than of pyogenic spondylitis. The presence of skip lesions and of a large paraspinal cold abscess is also suggestive of tuberculous spondylitis. However, because they barely penetrate the anterior longitudinal ligament, neither an anterior paraspinal phlegmon or an abscess encasing the intercostal arteries, is seen in spinal tuberculosis. MR imaging is less sensitive than radiography or CT for identifying paraspinal calcifications, which are a distinctive imaging feature of spinal tuberculosis. Pyogenic spondylitis most commonly involves the lumbar spine and one spinal segment.
How well did you know this?
1
Not at all
2
3
4
5
Perfectly
12
Q
  1. A CT cystogram is being performed on a 40-year-old man brought to the A&E department, after a fall from a roof. Blood is seen at the external urethral meatus on examination. The CT scan shows focal thickening of the urinary bladder wall, with no extravasation outside the bladder. Which of the following is the most likely injury sustained?
    A. Bladder contusion
    B. Intraperitoneal bladder rupture
    C. Extraperitoneal bladder rupture
    D. Combined intraperitoneal and extraperitoneal bladder rupture
    E. Subserosal bladder rupture
A
  1. A. Bladder contusion
    Bladder contusion is the most common bladder injury following trauma. Unlike the other choices there is no contrast extravasation. The CT scans can show an intramural haematoma (seen as a focal ellipse-shaped thickening of the bladder wall). This may appear as a crescent-shaped filling defect on cystography. Subserosal bladder rupture is seen as an elliptical contrast extravasation adjacent to the bladder on CT. Both intraperitoneal and extraperitoneal bladder rupture can be seen as extravasation outside the bladder.
How well did you know this?
1
Not at all
2
3
4
5
Perfectly
13
Q
  1. A 28-year-old man with sudden-onset of heart murmur and a normal chest radiograph is admitted for progressive shortness of breath on exertion over the recent months. There is a positive family history for heart murmurs and sudden death of a sibling at the age of 30 years. While in the hospital he developed an acute, severe bout of central chest pain, which prompted an urgent CT of the chest. The CT of the chest shows dissection of an enlarged ascending aorta. What is the diagnosis?
    A. Homocystinuria
    B. Marfan syndrome
    C. Ehlers-Danlos syndrome
    D. Pseudoxanthoma elasticum
    E. Mucopolysaccharidosis
A
  1. B. Marfan syndrome
    Annulo-aortic ectasia, a condition characterised by dilated sinuses of Valsalva with effacement of the sinotubular junction, with normal calibre arch, is most commonly associated with Marfan syndrome. Other causes include homocystinuria, Ehlers-Danlos syndrome and osteogenesis imperfecta; however, annulo-aortic ectasia can be idiopathic, although onset and progression is more rapid in Marfan syndrome. Common cardiovascular manifestations, include annulo-aortic ectasia with or without aortic valve insufficiency, aortic dissection, aortic aneurysm, pulmonary artery dilatation and mitral valve prolapse, most of which are substantial contributors to mortality.
    Homocystinuria presents with thromboembolic episodes like stroke. Murmurs related to AR or MR are not commonly associated with the other conditions.
How well did you know this?
1
Not at all
2
3
4
5
Perfectly
14
Q
  1. A 59-year-old patient is admitted with general lethargy, weight loss and gradual abdominal distension. Diagnostic work-up included an abdominal CT scan, which demonstrated thickening of the peritoneal surfaces and a large, multiloculated dense ascites, causing secondary scalloping of the liver edge. What is the most likely location of the primary tumour?
    A. Stomach
    B. Appendix
    C. Pancreas
    D. Liver
    E. Rectum
A
  1. B. Appendix
    The clinical picture and CT findings are characteristic for pseudomyxoma peritonei, a process of gradual accumulation of large amounts of gelatinous/mucinous material within the peritoneum. This accumulation of mucin is secondary to a ruptured mucocoele of the appendix. Characteristic CT findings include omental caking, thickening of the peritoneum and mesentery, large gelatinous ascites and scalloped contour of the liver and/or splenic borders. The ascites may vary from water density to a very thick soft-tissue density, which is dependent on the volume of mucin within the fluid. It is important to always scrutinise the appendix to identify the mucocoele once the secondary features are identified.
How well did you know this?
1
Not at all
2
3
4
5
Perfectly
15
Q
  1. A 44-year-old woman with recurrent urinary tract infections is referred for a renal tract ultrasound. This demonstrates normal kidneys and multiple fluid-filled cysts within the bladder wall. Which of the following is the most likely cause?
    A. Transitional cell carcinoma
    B. Cystitis cystica
    C. Emphysematous cystitis
    D. Eosinophilic cystitis
    E. Interstitial cystitis
A
  1. B. Cystitis cystica
    Cystitis cystica and cystitis glandularis are inflammatory processes of the bladder wall with multiple small, round, cyst-like elevations in the submucosa. They are often associated with irritants such as chronic infection, calculi or bladder outlet obstruction. The hallmark of emphysematous cystitis is gas within the bladder wall. The main risk factors are diabetes mellitus and bladder outflow obstruction. Eosinophilic cystitis can present with a nodular bladder wall, but the nodules would be echogenic on ultrasound in comparison with the fluid-filled cysts of cystitis cystica. In any event, all focal bladder abnormalities seen at imaging should be evaluated cystoscopically. In interstitial cystitis, the bladder wall becomes thick and trabeculated.
How well did you know this?
1
Not at all
2
3
4
5
Perfectly
16
Q
  1. A 22-year-old man presents to the A&E department with a painful swollen ankle following a twisting injury’. Plain X-rays showed no fracture, although diffuse soft-tissue swelling was evident. The ankle mortise was intact. Incidental note was made of a benign lesion in the mid- shaft of the fibula, which the reporting radiologist described as a fibrous cortical defect. Which one of the following statements regarding this entity is false?
    A. They are smaller than non-ossifying fibromas.
    B. Pathological fractures tend to end in non-union.
    C. Both show dense sclerotic border on CT.
    D. They commonly affect the metaphysis of long bones.
    E. They are uncommon in the upper extremity.
A
  1. B. Pathological fractures tend to end in non-union.
    Benign bone tumours in the fibrous group include benign fibrous cortical defect (FCD), non-ossifying fibroma (NOF), osteo-fibrous dysplasia (OFD), fibrous dysplasia (FD) and fibroma. Benign fibrous cortical defects and non-ossifying fibromas are the most common tumours in the benign fibrous group. Benign cortical defects and non ossifying fibromas are eccentric, cortical bone lesions, usually located in the metaphyses of long bones. Non-ossifying fibromas are usually larger than fibrous cortical defects. The most common locations for fibrous cortical defects are the distal femur, proximal tibia and distal tibia. They occur less commonly in the fibula and are relatively uncommon in the upper extremity.
    On CT, both benign fibrous cortical defects and non-ossifying fibromas have a dense sclerotic border, and on MRI both are of low signal intensity on T1 weighted and T2-weighted images.
    Pathologic fractures arc more common witli the larger non-ossifying fibroma variant. They tend to heal spontaneously.
How well did you know this?
1
Not at all
2
3
4
5
Perfectly
17
Q
  1. A 43-year-old man presents to the A&E department with severe headache and is sent for an urgent CT brain, which is normal. MRI shows loss of normal flow void in the basal cisterns with intense enhancement along the cisterns on post-contrast images. What is the likely diagnosis?
    A. Subarachnoid haemorrhage
    B. Lymphoma
    C. TB meningitis
    D. Ruptured dermoid cyst
    E. Creutzfeldt-Jakob disease (CJD)
A
  1. C. TB meningitis
    Tuberculous meningitis (TBM) is the most common manifestation of CNS tuberculosis across all age groups. The typical radiographic finding is abnormal meningeal enhancement, usually most pronounced in the basal cisterns. These findings are better seen at gadolinium-enhanced MR imaging than at CT. Appearances usually resolve relatively quickly with adequate treatment; however, radiographic resolution is delayed if there are thickened exudates. This appearance is non-specific and has a wide differential diagnosis that includes meningitis from other infective agents; inflammatory diseases such as rheumatoid arthritis and sarcoidosis; and neoplastic causes, both primary and secondary. The presence of high density within the basal cisterns on non-contrast CT scans is a very specific sign for TBM in children.
    The most common complication of tuberculous meningitis is communicating hydrocephalus. Ischaemic infarcts are also common, being seen in 20% 40% of patients at CT, mostly within the basal ganglia or internal capsule resulting from vascular compression and occlusion of small perforating vessels. Cranial nerve (2, 3, 4 and 7) involvement is reported.
How well did you know this?
1
Not at all
2
3
4
5
Perfectly
18
Q
  1. A 6-year-old child is admitted to the emergency department with a head injury. The emergency department consultant demands an urgent CT of the head. Which of the following risk factors would warrant a CT of the head within 1 hour of the injury?
    A. Two episodes of vomiting after the head injury
    B. Amnesia of events 10 minutes preceding the head injury
    C. Fall from a playground climbing frame of 3 metres in height
    D. GCS of 13 on initial clinical assessment
    E. Abnormal drowsiness
A
  1. D. GCS of 13 on initial clinical assessment
    According to the latest NICE guidelines, for children who have sustained a head injury’ and have any of the following risk factors, perform a CT head scan within 1 hour of the risk factor being identified;
    * Suspicion of non-accidental injury
    * Post-traumatic seizure but no history of epilepsy’
    * On initial emergency department assessment, GCS less than 14 or, for children under 1 year, GCS (paediatric) less than 15
    * At 2 hours after the injury’, GCS less than 15
    * Suspected open or depressed skull fracture or tense fontanelle
    * Any sign of basal skull fracture (haemotympanum, ‘panda’ eyes, cerebrospinal fluid leakage from the ear or nose, or Battle’s sign)
    * Focal neurological deficit
    * For children under 1 year, presence of bruise, swelling or laceration of more than 5 cm on the head
    For children who have sustained a head injury and have more than one of the following risk factors (and none of those mentioned above), perform a CT head scan within 1 hour of the risk factors being identified:
    * Loss of consciousness lasting more than 5 minutes (witnessed)
    * Abnormal drowsiness
    * Three or more discrete episodes of vomiting
    * Dangerous mechanism of injury (high-speed road traffic accident either as pedestrian, cyclist or vehicle occupant; fall from a height of greater than 3 metres; high speed injury from
    a projectile or other object)
    * Amnesia (antegrade or retrograde) lasting more than 5 minutes
How well did you know this?
1
Not at all
2
3
4
5
Perfectly
19
Q
  1. An 8-year-old boy attends the emergency department after his bicycle collides with an oncoming car. The ambulance crew have immobilised his cervical spine and report that the patient was complaining of severe neck ache. All of the following risk factors warrant a CT of the cervical spine without obtaining a plain radiograph, except
    A. The patient is already attending the department for a CT of the head.
    B. Fall from a height of more than 1 metre.
    C. Loss of sensation in the upper arms.
    D. GCS of 13 on initial assessment.
    E. The child needs to be intubated.
A
  1. B. Fall from a height of more than 1 metre According to the latest NICE guideline for spinal injury:
    Perform MRI for children (under 16s) if there is a strong suspicion of
    * Cervical spinal cord injury as indicated by the Canadian C-spine rule and by clinical assessment or
    * Cervical spinal column injury as indicated by clinical assessment or abnormal neurological signs or symptoms, or both.
    Consider plain X-rays in children (under 16s) who do not fulfil the criteria for MRI in recommendation but clinical suspicion remains after clinical assessment.
    For imaging in children (under 16s) with head injury and suspected cervical spine injury, follow the recommendations in the NICE guideline on head injury.
    For children with a head injury, perform a CT cervical spine scan if
    * GCS is less than 13 on initial assessment
    * The patient has been intubated.
    * Focal peripheral neurological signs.
    * Paraesthesia in the upper or lower limbs.
    * A definitive diagnosis is needed urgently (e.g. before surgery).
    * The patient is having CT for head injury or multiregion trauma.
    * There is strong clinical suspicion of injury despite normal X-rays.
    * Plain X rays are technically difficult or inadequate.
    * Plain X-rays identify a significant bony injury.
    CT is to be performed within 1 hour of the risk factor being identified.
How well did you know this?
1
Not at all
2
3
4
5
Perfectly
20
Q
  1. A 34-year-old man with history of measles infection showed marked cerebral atrophy on MRI with high signal in the deep white matter bilaterally on T2 and FLAIR images. What is the most likely diagnosis?
    A. Adrenoleukodystrophy
    B. Alexander s disease
    C. CJD
    D. Progressive multifocal leukoencephalopathy (PML)
    E. Subacute sclerosing panencephalitis
A
  1. E. Subacute sclerosing pan encephalitis
    Creutzfeldt-Jakob disease (CJD) shows high signal intensities in the basal ganglia (putamen and caudate nucleus) and in the cortex on DW images. The high signal intensities in the basal ganglia are also prevalent on T2-weighted and FLAIR images. The cortical hyperintensities are usually not visualised on T2-weighted and FLAIR images (advantage of DW imaging).
    Progressive multifocal leukoencephalopathy (PML) is a demyelinating disease of immunocompromised patients caused by human papovaviruses. Subacute sclerosing panencephalitis (SSPE) occurs several years after measles infection. SSPE typically starts with mental and behavioural abnormalities, myoclonia, tremor and seizures. Multifocal, hyperintense foci in white matter and the basal ganglia have been reported in PML and SSPE on T2-weighted images. On T2-weighted and FLAIR images, PML and SSPE are associated with white matter lesions, whereas CJD is not. The high-signal-intensity cortical lesions on DW images may be also a hallmark of CJD.
How well did you know this?
1
Not at all
2
3
4
5
Perfectly
21
Q
  1. A 25-year-old man presents with left hip/groin pain after exercise that worsens on internal rotation of the hip. A plain AP radiograph of the pelvis shows an osseous protrusion at the femoral head-neck junction, and the measured alpha angle is greater than 55 degrees. Which of the following is the most likely diagnosis?
    A. Pincer type femoro-acetabular impingement
    B. Cam type femoro-acetabular impingement
    C. Missed congenital hip dislocation
    D. Focal acetabular over-coverage
    E. Protrusio acetabuli
A
  1. B. Cam-type femoro-acetabular impingement
    Femoro-acetabular impingement (FAT) is a major cause of premature osteoarthritis of the hip. It is split radiographically into two main types. Pincer-type impingement is the acetabular cause of FA I and is secondary to either focal or generalised over-coverage of the femoral head by the acetabulum. Cam-type impingement is the femoral cause and is secondary to an asphericity of the femoral head and offset of the femoral head-neck junction. Cam impingement is more common in young men and leads to a increased alpha angle (diagnostic > 55 degrees). The osseous bump at the head-neck junction may be located laterally, resulting in the ‘pistol grip’ deformity seen on AP pelvis.
    Protrusio acetabuli occurs when the femoral head is overlapping the ilioischial line medially and may be idiopathic or secondary to causes such as rheumatoid arthritis, Paget disease Marians syndrome and osteomalacia.
How well did you know this?
1
Not at all
2
3
4
5
Perfectly
22
Q
  1. Chest radiograph of a currently asymptomatic 84-year-old man shows a large well-defined soft tissue density mass in the left apex with a sharp inferior margin. There is underlying rib abnormality, suggesting previous surgery and sheet-like pleural calcification in the left mid and lower zone. What is the diagnosis?
    A. Aspergilloma
    B. Plombage
    C. Pancoast tumour
    D. Bronchogenic cyst
    E. Lymphoma
A
  1. B. Plombage
    In the pre chemotherapy era, surgical management of pulmonary TB included thoracoplasty or plombage, in which an extrapleural space was created between parietal pleura and the chest wall, which was filled with materials such as fat, oil, wax packs, bone or methyl methacrylate (Incite) balls.
    Appearance on chest radiograph depend on the material used, with associated chest wall deformity, resected ribs or stigma of pulmonary TB-like calcified granulomas, lymph nodes or sheets of pleural calcification.
    Pancoast tumour when large would often be associated with destroyed ribs, and the description does not fit with an aspergilloma (visible air crescent) or bronchogenic cysts, which are homogenous and mostly located around the carina.
How well did you know this?
1
Not at all
2
3
4
5
Perfectly
23
Q
  1. An adult patient was admitted to hospital with abdominal pain, jaundice and a palpable epigastric mass. Ultrasound demonstrated isolated dilatation of the common bile duct with otherwise normal appearance of the proximal biliary tree. What is the most likely diagnosis based on the sonographic findings?
    A. Choledochal cyst
    B. Caroli disease
    C. Choledochocoele
    D. Common bile duct diverticulum
    E. Impacted common bile duct calculus
A
  1. A. Choledochal cyst
    The Todani classification system is used to differentiate the cystic processes of the biliary tree into five groups. Type 1 cysts, known as choledochal cysts, are responsible for 90% of cystic biliary disease. These cysts are further subclassified into LA (dilation of the entire extrahepatic bile duct), IB (focal segmental dilation of the extrahepatic duct) and IC (dilation only affecting the common duct). Patients may present with the triad of vague abdominal pain, jaundice and a palpable epigastric mass, although this is only reported in 10%-20% of patients. Type 2 ‘cysts’ are true diverticulae of the bile duct. Type 3, known as a choledochocoele, is a focal protrusion of CBD into the duodenum. Type 4, consists of multiple communicating ultra and extra hepatic duct cysts. Type 5, known as Caroli’s disease, represents cystic dilatation of intra-hepatic ducts.
How well did you know this?
1
Not at all
2
3
4
5
Perfectly
24
Q
  1. A 40-year-old patient emigrating from an African country is investigated for stone disease because of left-sided renal angle pain. CT of the kidneys, ureters and bladder (KUB) confirms a left renal calculus but also shows thin curvilinear calcification outlining a normal-sized bladder with involvement of the distal ureters only. Which of the following is the most likely cause?
    A. Tuberculosis
    B. Escherichia coli infection
    C. Transitional cell carcinoma
    D. Malakopiakia
    E. Schistosomiasis
A
  1. E. Schistosomiasis
    This is a classical description of schistosomiasis infection of the bladder, with curvilinear calcification spreading proximally from the bladder into the distal ureters. The bladder wall can otherwise appear normal but is often thick-walled or nodular. In tuberculosis, the calcification starts in the kidney and can then extend more distally. When involved, the bladder is usually contracted, rather than normal size. Escherichia coli infection is associated with emphysematous cystitis; bacterial infection can be associated with bladder calculi but not mural calcification. Calcification of transitional cell carcinoma can be linear, curvilinear or stippled; however, a mass or wall thickening would be expected and 97% of cases occur in over 45-year-olds. Malakoplakia is a rare condition and calcification in affected patients is uncommon.
How well did you know this?
1
Not at all
2
3
4
5
Perfectly
25
Q
  1. An asymptomatic military recruit showed a well-defined 2 x 2 cm solitary pulmonary nodule in the right lower zone on a plain chest radiograph. On dose inspection, the nodule showed faint calcification. CT performed for further characterisation showed areas of fat density on pixelometry. What is the diagnosis?
    A. Tuberculoma
    B. Aspergilloma
    C. Hamartoma
    D. Carcinoid
    E. Haemangioma
A
  1. C. Hamartoma
    Hamartoma is a benign neoplasm composed of mesenchymal tissues such as cartilage, fat, connective tissue, smooth muscle and calcification. Hamartoma are common benign tumours. Typical CT findings consist of a well-defined, smooth, round or lobulated nodule or mass with fat density in nearly two-thirds of cases, and popcorn like calcification or central calcification in a quarter.
How well did you know this?
1
Not at all
2
3
4
5
Perfectly
26
Q
  1. You are performing an abdominal ultrasound scan on a woman who has been complaining of chronic abdominal pain. There is a large 20 cm multiloculated, ovoid anechoic mass in the right lobe of liver. The internal septations are well visualised and hyperechoic. Further investigation with CT demonstrates enhancement of its thick wall and internal septations. What is the most likely diagnosis?
    A. Simple hepatic cyst
    B. Choledochal cyst
    C. Echinococcal cyst
    D. Caroli disease
    E. Biliary cystadenoma
A
  1. E. Biliary cystadenoma
    Biliary cystadenomas are benign neoplasms originating in bile ducts, most commonly seen within the right lobe of liver. They rarely affect the extrahepatic biliary system. Women are affected more than men and typically present with chronic, vague abdominal pain. Biliary cystadenomas can reach up to 35 cm in size and are usually demonstrated as large multiloculated anechoic masses on ultrasound. A CT scan typically demonstrates a water density mass that shows enhancement of its walls and septations following administration of contrast, differentiating these from simple cysts. An echinococcal cyst (hydatid cyst) is a parasitic infection causing cystic transformation in the liver
How well did you know this?
1
Not at all
2
3
4
5
Perfectly
27
Q
  1. An intravenous urogram is performed on a 70-year-old diabetic for recurrent E. coli urinary tract infections. This demonstrates multiple small, smooth, plaque-like mural defects of
    the bladder and distal ureteric walls. Which of the following is the most likely cause?
    A. Emphysematous cystitis
    B. Malakoplakia
    C. Leukoplakia
    D. Haemorrhagic cystitis
    E. Bladder outflow obstruction
A
  1. B. Malakoplakia
    Malakoplakia is a rare chronic inflammatory disease primarily affecting the bladder, with decreasing incidence with increasing proximity to the kidney. Small smooth papules, nodules or plaque-like mural defects of the affected urinary tract are characteristic. Malakoplakia can be associated with immunosuppression, diabetes mellitus, renal transplants or long-term corticosteroid use. Leukoplakia is characterised by soft-tissue flakes that can be passed in the urine during micturition. Haemorrhagic cystitis and emphysematous cystitis can occur with E. coli infections but are associated with intraluminal blood clots and gas deposits, respectively. Bladder outflow obstruction results in a thick walled bladder with trabeculations.
How well did you know this?
1
Not at all
2
3
4
5
Perfectly
28
Q
  1. A 45 year-old woman presents with right hip pain. An AP radiograph of the hip shows a large lucent lesion with stippled calcification and a wide destructive-appearing zone of transition. Which of the following is the most likely diagnosis?
    A. Osteomyelitis
    B. Osteosarcoma
    C. Chondrosarcoma
    D. Fibrous dysplasia
    E. Chondromyxoid fibroma
A
  1. C. Chondrosarcoma
    The radiographic description is aggressive and the stippled calcification is suggestive of a chondroid matrix. The most likely diagnosis is therefore a chondrosarcoma. These may be primary or develop from an enchondroma or osteochondroma.
    Although chondromyxoid fibroma would be a possibility in this age group, it is an exceedingly rare benign chondral lesion that would normally have well-defined, thickened sclerotic margins.
    Chondrosarcomas are quite common tumours that may have relatively benign radiographic appearances. They are normally metaphyseal, located centrally within the skeleton and may exhibit endosteal scalloping. The diagnosis should also be considered in patients with increased pain or growth in centrally located enchondromas or osteochondromas. Surgical resection is the treatment of choice and the 5-year survival rate is approximately 75%.
How well did you know this?
1
Not at all
2
3
4
5
Perfectly
29
Q
  1. A 37-year-old woman with progressive worsening of headache had a facial series done following an injury to the face when she fell while skating. PA view of the skull and OM view did not show any fractures. Incidental note was made of enlarged optic canals. All of the following are associated with this, except
    A. NF1
    B. Optic glioma
    C. Sarcoidosis
    D. Sphenoid mucocoele
    E. Ophthalmic artery aneurysm
A
  1. C. Sarcoidosis
    Causes of optic canal enlargement (more than 6.5 mm in diameter)
    Common causes:
    * Glioma of the optic nerve
    * Meningioma of the optic nerve sheath
    * Metastasis
    * Neurofibromatosis with or without optic neurofibroma or glioma

Uncommon causes:
* Aneurysm of the ophthalmic artery or cavernous portion of internal carotid artery
* Arteriovenous malformation with ophthalmic artery involvement
* Carcinoma of ethmoid or sphenoid sinus
* Granuloma (e.g. tuberculosis, sarcoidosis)
* Increased intracranial pressure
* Mucocoele of sphenoid sinus
* Mucopolysaccharidoses (especially Hurler syndrome)
* Pituitary adenoma or craniopharyngioma extending anteriorly
* Pseudotumour of orbit
* Retinoblastoma with intracranial extension

How well did you know this?
1
Not at all
2
3
4
5
Perfectly
30
Q
  1. A 1 -year-old child is admitted with seizures and found to have bilateral subdural haematomas. There is concern regarding non accidental injury and a full skeletal survey is requested. According to the Royal (College of Radiologists guidelines, which radiograph(s) would not be routinely included in a full skeletal survey?
    A. PA radiograph of the right hand
    B. AP radiograph of the left tibia and fibula
    C. AP radiograph of the cervical spine
    D. Lateral radiograph of the lumbar spine
    E. Right oblique radiograph of the chest
A
  1. C. AP radiograph of the cervical spine
    The standard child protection skeletal survey for suspected non accidental injury includes the following:
    Skull
    * AP, lateral and Townes view (the latter if clinically indicated).
    * Skull X-rays should be taken even if a CT scan has been or will be performed.
    Chest
    * AP including the clavicles.
    * Two oblique views to show ribs (‘left and right oblique’).
    Abdomen
    * AP including the pelvis and hips.
    Spine
    * Lateral: may require separate exposures of the cervical, thoracic and thoraco-lumbar regions.
    * AP views of the cervical spine are rarely diagnostic at this age and should only be performed at the discretion of the radiologist.
    Limbs
    * AP of both upper arms
    * AP of both forearms
    * AP of both femurs
    * AP of both lower legs
    * PA of hands
    * DP of feet
    Where an abnormality is suspected, these views should be supplemented by the following:
    * Lateral views of any suspected shaft fracture
    * AP and lateral coned views when a fracture is suspected
How well did you know this?
1
Not at all
2
3
4
5
Perfectly
31
Q
  1. A term neonate is found in respiratory distress with shortness of breath and tachypnoea on Day 1 of life. An urgent chest radiograph demonstrates a right sided pneumothorax.
    There is no history of any antenatal complications. What is the most likely underlying cause?
    A. Respiratory distress syndrome (pulmonary surfactant deficiency)
    B. Group B streptococcus infection
    C. Congenital pulmonary airway malformation (CPAM)
    D. Meconium aspiration
    E. Congenital diaphragmatic hernia (CDH)
A
  1. D. Meconium aspiration
    Meconium aspiration is the most common cause of neonatal respiratory distress in full-term or post-mature infants (hyaline membrane disease is the most common cause in premature infants).
    Chest X-ray shows the following:
    * Bilateral diffuse grossly patchy opacities (atelectasis + consolidation). Unlike hyaline membrane disease, air bronchogram is not a typical feature.
    * Hyperinflation with areas of emphysema (air trapping).
    * Spontaneous pneumothorax + pneumomediastinum (25%) requiring no therapy.
    * Small pleural effusions (20%).
    * Rapid clearing, usually within 48 hours.
How well did you know this?
1
Not at all
2
3
4
5
Perfectly
32
Q
  1. A 58-year-old man who worked in the mines for several years presented with progressive shortness of breath on exertion. Extensive interstitial thickening and small nodules bilaterally with large masses of consolidation in the upper lobes were noted on the most recent chest radiograph. Comparison with previous films suggested central migration of the consolidation like upper lobar masses. What is the diagnosis?
    A. Pneumoconiosis with progressive massive fibrosis
    B. End-stage sarcoidosis
    C. End stage Langerhans Cell Histiocytosis (LCH)
    D. Cryptogenic fibrosing alveolitis
    E. Old TB
A
  1. A. Pneumoconiosis with progressive massive fibrosis
    On chest radiographs, large opacities (progressive massive fibrosis) may be seen in complicated coal worker pneumoconiosis, as in complicated silicosis.
    The large opacities result from nodule coalescence and are observed commonly in the middle lung zone or peripheral one-third of the lung on axial chest images and in the upper lung zone on longitudinal images. The large opacities gradually migrate towards the hilum, leaving emphysematous lung tissue between the fibrotic tissue and the pleural surface.
How well did you know this?
1
Not at all
2
3
4
5
Perfectly
33
Q
  1. A patient is admitted to hospital with progressively worsening jaundice and raised bilirubin levels. Ultrasound of the abdomen, demonstrates a large hyperechoic focus casting a shadow within the neck of the gallbladder, causing secondary dilation of the common hepatic and intrahepatic bile ducts proximally. The distal common bile duct (CBD) was normal.
    What is the most likely diagnosis?
    A. Caroli disease
    B. Choledochocoele
    C. Caroli syndrome
    D. Mirizzi syndrome
    E. Cholangiocarcinoma
A
  1. D. Mirizzi syndrome
    The sonographic findings of an impacted gallstone within the neck of bladder causing proximal biliary dilatation are diagnostic for Mirizzi syndrome. Mirizzi syndrome is caused by impaction of a large gallstone in the cystic duct, cystic duct remnant or gallbladder neck. The impacted stone causes external compression of the CBD, resulting in proximal dilation of the biliary tree. A choledochocoele is cystic dilatation of the CBD within the lumen of the duodenum. Caroli disease is a congenital disorder characterised by saccular dilatations of intrahepatic ducts. Caroli disease, in association with congenital hepatic fibrosis, polycystic kidney disease and others, is known as Caroli syndrome.
How well did you know this?
1
Not at all
2
3
4
5
Perfectly
34
Q
  1. A 30-year-old man is admitted with lower abdominal pain following a road traffic accident in which he was an unrestrained passenger. Blood is seen in his urine and a bladder injury is suspected. Regarding extraperitoneal bladder rupture, which of the following is incorrect?
    A. It is more common than intraperitoneal rupture.
    B. A flame shaped extravasation of contrast can often be seen.
    C. It is usually caused by puncture from a pelvic fracture.
    D. Contrast most commonly extravasates into the retropubic space of Rctzius.
    E. The bladder dome is the most common site of injury.
A
  1. E. The bladder dome is the most common site of injury.
    Extraperitoneal bladder rupture accounts for 80% of bladder ruptures. A flame-shaped extravasation of contrast is a classic finding and can be seen to extend into the perivesical fat and into the retropubic space of Retzius, anterior abdominal wall, upper thigh or scrotum.
    The most common site of injury is close to the anterolateral aspect of the bladder base. It us usually caused by puncture from pelvic fractures.
How well did you know this?
1
Not at all
2
3
4
5
Perfectly
35
Q
  1. A 4 year-old child presents with short stature and failure to grow. Plain radiographs reveal multiple abnormalities, including generalised increased density of long bones with thickened cortices, widened cranial sutures, Wormian bones, a hypoplastic mandible and shortened pointed distal phalanges. Which of the following is the most likely diagnosis?
    A. Pyknodysostosis
    B. Osteopetrosis
    C. Cleidocranial dysostosis
    D. Osteosclerosis
    E. Kinky hair syndrome
A
  1. A. Pyknodysostosis
    Pyknodysostosis is a congenital abnormality that should be considered in the differential diagnosis of osteosclerosis. The patients are typically short, have hypoplastic mandibles, widened cranial sutures, Wormian bones, brachycephaly, clavicular dysplasia, thick skull base and hypoplasia or non pneumatisation of the paranasal sinuses. The distinguishing feature is aero-osteolysis with sclerosis. The distal phalanges appear as if they have been put in a pencil sharpener - they are pointed and dense.
How well did you know this?
1
Not at all
2
3
4
5
Perfectly
36
Q
  1. A man with nasopharyngeal squamous cell carcinoma undergoes MRI staging. A pathological node is seen lateral to the IJV. It lies below’ the hyoid bone but above the vocal cords. At which level is this likely to be located?
    A. I
    B. II
    c. III
    D. IV
    E. V
A
  1. C. Ill
    Clinical classification of neck nodes Level I: Above hyoid bone Level Ia: Previously called submental nodes Level Ib: Previously called submandibular nodes Level II: from skull base to level of lower body of hyoid bone Posterior to back of submandibular gland Anterior to back of sternocleidomastoid muscle Level IIA: anterior, lateral, medial or posterior to internal jugular vein Inseparable from internal jugular vein (if posterior to vein)
    Previously classified as upper internal jugular nodes Level IIB: Posterior to internal jugular vein with fat plane separating nodes and vein previously classified as upper spinal accessory nodes
    Level III: Prom level of lower body of hyoid bone to level of lower cricoid cartilage arch, anterior to back of sternocleidomastoid muscle, previously known as mid-jugular nodes.
    Level IV: From level of lower cricoid cartilage arch to level of clavicle
    Level V: Posterior to back of sternocleidomastoid muscle from skull base to level of lower cricoid arch Level VI: Between carotid arteries from level of lower body of hyoid bone to level superior to top of manubrium
    Level VII: Between carotid arteries below level of top of manubrium
How well did you know this?
1
Not at all
2
3
4
5
Perfectly
37
Q
  1. CT of the chest in a 68-year-old man who worked for 20 years in a coal mine, showed an ill defined 3-cm spiculated nodule in the apical segment of the lower lobe of the left lung. The lesion was avid on PET and showed high signal on axial T2W MRI thorax.
    All of the following are true regarding lung cancer and progressive massive fibrosis except:
    A. T2W is useful in differentiating lung cancer from PMF.
    B. PET CT is useful in differentiating lung cancer from PMF.
    C. PMF shows low signal compared to muscle on T1W MRI.
    D. PMF commonly shows peripheral enhancement on contrast MRI.
    E. Histopathologic analysis should still be performed for diagnosis.
A
  1. B. PET CT is useful in differentiating lung cancer from PMP.
    It is clinically and radiologically important to differentiate progressive massive fibrosis from lung cancer. MR! is a preferable option because lung cancer appears as high signal lesion on T2 weighted images, whereas progressive massive fibrosis appears as a low-signal abnormality when compared with the signal of muscle on both Tl weighted and T2-weighted images. On post-contrast MR, progressive massive fibrosis appears peripherally enhanced more frequently than not. Histopathologic analysis should still be performed for diagnosis.
    PET may show intensive uptake of FDG in the fibrotic mass in progressive massive fibrosis; hence differentiation from cancer may be difficult.
How well did you know this?
1
Not at all
2
3
4
5
Perfectly
38
Q
  1. A patient is admitted to hospital with jaundice and recent weight loss. Abdominal ultrasound demonstrates marked dilation of the intrahepatic ducts, extending to the liver surface. Note is also made of an isocchoic mass at the porta hepatis. Which of the following is the most likely diagnosis?
    A. Primary sclerosing cholangitis R. Klatskin tumour
    C. Mirizzi syndrome
    D. Choledochal cyst
    E. Caroli disease
A
  1. B. Klatskin tumour
    Klatskin tumours are cholangiocarcinomas that involve the confluence of hepatic ducts and account for up to 70% of cholangiocarcinomas. Dilation of the intrahepatic ducts is the most frequent abnormal finding on ultrasound. Lobar atrophy may be very subtle on ultrasound but may be identified by secondary findings such as extension of dilated ducts to the liver surface and lobar ductal crowding. The tumour itself may be very’ difficult to identify on ultrasound but may present as an isoechoic or hyperechoic mass. Primary sclerosing cholangitis is a fibrotic process that affects the biliary tree causing multifocal strictures and may lead to biliary cirrhosis. Mirizzi syndrome is dilation of the proximal common bile duct and intrahepatic ducts secondary’ to external compression of a large gallstone impacted in the cystic duct/gallbladder neck.
How well did you know this?
1
Not at all
2
3
4
5
Perfectly
39
Q
  1. A 17 year-old girl is admitted following a road traffic accident where she was a restrained passenger. She has low-volume haematuria following catheterisation. Pre- and post-intravenous (IV) contrast CT is performed after administering contrast via her urinary catheter. The pre contrast CT shows contrast within the peritoneal cavity around loops of bowel and there is irregularity of the bladder wall at the dome of the bladder. No contrast blushes are seen on the arterial-phase CT. Which of the following is the most likely injury?
    A. Extraperitoneal bladder rupture
    B. Intraperitoneal bladder rupture
    C. Subserosal bladder rupture
    D. Iliac artery injury with haemorrhage
    E. Pelvic haematuria from occult fracture
A
  1. B. Intraperitoneal bladder rupture
    Intraperitoneal bladder rupture occurs in around 20% of bladder ruptures. It can be caused by a sudden increase in intravesical pressure owing to blunt trauma and usually causes rupture to the bladder dome. Contrast can be seen in the peritoneal cavity, often outlining loops of bowel and
    extending up the paracolic gutters. Extraperitoneal bladder rupture is more common and is usually caused by puncture from pelvic fractures. Contrast can be seen in the retroperitoneal space, anterior abdominal wall, upper thighs or even the scrotum. It is important to distinguish between intraperitoneal and extraperitoneal rupture: intraperitoneal rupture requires surgical repair, whereas extraperitoneal rupture is managed conservatively. Contrast outside the bladder would not be seen on the pre IV contrast CT in cases of arterial damage or pelvic haematoma.
How well did you know this?
1
Not at all
2
3
4
5
Perfectly
40
Q
  1. A 65-year-old man presents with severe upper thoracic back pain, of insidious onset, with little relief from analgesia. Lateral radiographs of his thoracic spine reveals uniformly increased density within the T7 and T9 vertebral bodies, with retention of the vertebral body size and contour. Which of the following is the most likely cause for this finding?
    A. Bone metastasis
    B. Osteoid osteoma
    C. Tuberous sclerosis
    D. Osteopetrosis
    E. Fluorosis
A
  1. A. Bone metastasis
    The ‘ivory vertebra’ sign, as described in this case, refers to a diffuse and homogeneous increase in the density of a vertebral body, which retains its size and contours. It can occur in both adults and children but is more common in the former. In adults, ivory vertebra has been associated most commonly with fractures (compression or healing), haemangiomas, lymphoma, myelosclerosis, with metastatic disease (especially prostate, breast and carcinoid), chronic sclerosing osteomyelitis, Paget disease and renal osteodystrophy. An ivory vertebra at one or more vertebral levels in an elderly man is most compatible with a diagnosis of metastatic disease, commonly as the result of prostate carcinoma. The other options are all much less common causes of ivory vertebrae.
How well did you know this?
1
Not at all
2
3
4
5
Perfectly
41
Q
  1. A 40-year-old man was punched in the face during an altercation. Clinical examination showed left facial swelling with left-sided ophthalmoplegia and diplopia. X-rays showed a left maxillary fracture. What is the likely cause of the patient’s symptoms?
    A. Superior orbital fissure syndrome
    B. Cavernous sinus thrombosis
    C. Carotico-cavernous fistula
    D. Post-traumatic ophthalmoplegia
    E. Cerebral venous sinus thrombosis
A
  1. A. Superior orbital fissure syndrome
    Because the upper transverse maxillary buttress forms the orbital floor, fractures of this buttress may cause various orbital complications, including inferior rectus muscle tears, globe rupture or impingement, optic nerve injury and orbital hematoma.
    Superior orbital fissure syndrome is caused by extension of the fracture through the superior orbital fissure, with resultant injury to cranial nerves III, IV, VI (the ophthalmic branch of the trigeminal nerve) and VI as they traverse the fissure into the orbit, thus causing ophthalmoplegia or diplopia (extra-ocular muscle paralysis) and ptosis (paralysis of the levator palpebrae superioris, which is supplied by cranial nerve III). Additional injury to the optic nerve (cranial nerve II) at the orbital apex results in orbital apex syndrome, with unilocular visual loss added to the list of signs and symptoms. Orbital apex syndrome is a surgical emergency because prompt intervention is necessary to prevent permanent blindness.
How well did you know this?
1
Not at all
2
3
4
5
Perfectly
42
Q
  1. While reporting a plain radiograph of a 10 year-old girl patient’s left hand, you notice that her bone age is delayed. There is no indication of the patient’s underlying condition on the request form. The bones of the hand are otherwise unremarkable.
    Which of the following conditions may offer a possible explanation?
    A. Hypothyroidism
    B. Achondroplasia
    C. Haemophilia
    D. Cushing syndrome
    E. Precocious puberty
A
  1. A. Hypothyroidism
    Causes of delayed bone age include the following:
    * Constitutional: Familial and IUGR
    * Metabolic: Hypothyroidism, hypopituitarism, hypogonadism (Turner syndrome), Cushing discase/steroid therapy, diabetes mellitus, rickets and malnutrition
    * Systemic disease: Congenital heart disease, renal disease, coeliac disease, Crohn’s disease, ulcerative colitis and anaemia
    * Syndromes: Trisomies, Noonan disease, Cornelia de Lange, cleidocranial dysplasia, Lesch-Nyhan disease metatrophic dwarfism
    Of these, in the absence of any other history or radiological abnormality, hypothyroidism is more likely until proven otherwise. Precocious puberty results in advanced bone age.
How well did you know this?
1
Not at all
2
3
4
5
Perfectly
43
Q
  1. A 42 year old man with cough and haemoptysis, swelling of ankles and hand, raised urea, and creatinine on recent blood biochemical profile, presented to the A&E department with bloody nasal discharge. A Chest radiograph was performed. What do you expect the chest radiograph to show given the clinical scenario?
    A. Miliary nodules
    B. Pew varying sized nodules, some with cavitation
    C. Perihilar infiltrates in a batwing pattern
    D. Bilateral upper lobe fibrosis
    E. Tramline and gloved finger opacities
A
  1. B. Few varying sized nodules, some with cavitation
    Wegener’s granulomatosis, currently called granulomatosis with polyangitis, is a probable autoimmune disease characterised by systemic necrotising granulomatous destructive angitis, with a classic triad of respiratory tract inflammation, systemic small vessel vasculitis and necrotising glomerulonephritis.
    Sinus disease classically includes destructive sinusitis, nasal septal ulceration, septal perforation and saddle nose deformity.
    Chest radiograph shows interstitial fibrosis at the bases, which is usually asymptomatic. Multiple pulmonary nodules with cavitation are the most common and characteristic manifestation. Pleural effusions and mediastinal nodal enlargement are seen.
    There are multiple causes of miliary nodules, described as innumerable small 1-4 mm lung nodules, described in miliary tuberculosis, fungal infection, metastasis (thyroid, renal, breast, melanoma, pancreas, trophoblastic tumour), sarcoidosis, pneumoconiosis, pulmonary haemosiderosis, varicella infection, hypersensitive pneumonitis, histiocytosis etc. Bilateral upper lobe fibrosis are classically described in TB, ankylosing spondylitis, sarcoidosis, silicosis, radiation and histiocytosis.
How well did you know this?
1
Not at all
2
3
4
5
Perfectly
44
Q
  1. An 87 year old female patient from a nursing home presents with a history of severe abdominal pain and diarrhoea. She has a past medical history of transient ischaemic attacks and two non-ST elevation myocardial infarctions. Blood results show an elevated lactate level and raised white cell count. CT report states that the most likely cause is ischaemic colitis. What is the most likely CT finding?
    A. Thick-walled caecum with distended appendix
    B. Dilated loop of bowel in the left iliac fossa with spiralling vessels
    C. Thick-walled oedematous loops of bowel with branching lucencies peripherally in liver
    D. Thick-walled oedematous colon with ascites
    E. Dilated small bowel loops with branching linear lucency at the porta hepatis
A
  1. C. Thick-walled oedematous loops of bowel with branching lucencies peripherally in the liver The patient in this scenario has a history of atherosclerosis with end organ damage. This would make
    them at risk of developing ischaemic colitis. The area of bowel affected in this question is a watershed area and is most susceptible to ischaemic colitis. The other plain film findings described in this question include intramural gas and portal venous gas. These are very worrying signs and usually imply a poor outcome. Sigmoid volvulus would cause a coffee bean sign on plain film. The findings are also not suggestive of caecal volvulus or small bowel obstruction. Necrotising enterocolitis would cause intramural and portal venous gas; however, this is a condition of neonates. Dilated small bowel with gas at the porta hepatis, is classic description of gall stone ileus, with gas being in the biliary tree.
How well did you know this?
1
Not at all
2
3
4
5
Perfectly
45
Q
  1. A 60-year-old male smoker attends for a CT scan following an ultrasound scan that showed unilateral hydronephrosis. The scan shows a lobulated irregular bladder wall mass that enhances in the portal venous phase, to a greater extent than the bladder wall. Which of the following is the likely diagnosis?
    A. Squamous cell carcinoma
    B. Adenocarcinoma
    C. Transitional cell carcinoma
    D. Leiomyoma
    E. Blood clot
A
  1. C. Transitional cell carcinoma
    Transitional cell carcinoma of the bladder is the most common urinary tract malignancy in the UK and appears as a lobulated, irregular bladder wall thickening that enhances more greatly than the normal bladder wall. If hydronephrosis is present, this suggests muscle invasive disease. Squamous cell carcinoma can look like this but accounts for only 4% of all bladder malignancies and is therefore less likely unless there are specific risk factors such as schistosomiasis infection, chronic cystitis, bladder calculi or long term catheterisation. Adenocarcinoma Is rare and associated with urachal remnants or bladder extrophy. Leiomyomas appears as smooth bladder wall masses. Blood clots do not enhance and are often seen to change in size and position on sequential scans.
How well did you know this?
1
Not at all
2
3
4
5
Perfectly
46
Q
  1. A 40-year-old man had a cardiac CT and MR! to evaluate suspected cardiomyopathy. This revealed a well-defined incidental lesion within the T10 vertebral body, with coarse vertical trabeculae on CT, and high signal on both T1 and T2 sequences on MRI. The lesion enhanced with contrast on both modalities. What is the most likely diagnosis?
    A. Plasmacytoma
    B. Osteosarcoma
    C. Osteopathia striata
    D. Enostosis
    E. Haemangioma
A
  1. E. Haemangioma
    Vertebral hemangiomas are present in 5%-11% of all autopsies and are multiple in one-third of these cases. Lesions commonly occur in the vertebral bodies of the lower thoracic and lumbar spine, and radiographic appearances are those of coarse vertical trabeculations, giving a ‘corduroy’ or ‘honeycomb’ appearance. A CT scan may also demonstrate small punctuate areas of sclerosis, giving a ‘polka-dot’ appearance. On MRI, the lesions are of a mottled pattern with a characteristic low to high signal on T1-weighted sequences and high signal on T2-weighted sequences in proportion to the amount of adipose tissue. The lesions enhance strongly on both CT and MRI because of hypervascularity.
How well did you know this?
1
Not at all
2
3
4
5
Perfectly
47
Q
  1. A 30-year-old woman presents with a painless neck mass. MRI confirms that this is a thin-walled cyst lying just anterior to the sternocleidomastoid. There is a tract from the bifurcation of the common carotid artery. What is the diagnosis?
    A. Lymphangioma
    B. Thyroglossal duct cyst
    C. First branchial cleft cyst
    D. Carotid body tumour
    E. Second branchial cleft cyst
A
  1. E. Second branchial cleft cyst
    The vast majority (95%) of branchial cleft anomalies arise from the second cleft. At least three-fourths of these anomalies are cysts, which typically occur between 10 and 40 years of age, in contrast to fistulas or sinuses, which manifest most commonly during the first decade of life. No gender predilection has been reported.
    These cysts usually appear as painless, fluctuant masses in the lateral portion of the neck adjacent to the anteromedial border of the sternocleidomastoid muscle at the mandibular angle. The mass enlarges slowly over time and may become painful and tender if secondarily infected.
    At US, a second branchial cleft cyst is seen as a sharply marginated, round to ovoid, centrally anechoic mass with a thin peripheral wall that displaces the surrounding soft tissues. The mass is compressible and shows distinct acoustic enhancement. Occasionally, fine, indistinct internal echoes, representing debris, may be seen. The ‘classic’ location of these cysts (at either CT or MR imaging) is at the anteromedial border of the sternocleidomastoid muscle, lateral to the carotid space and at the posterior margin of the submandibular gland. The cyst typically displaces the sternocleidomastoid muscle posteriorly or posterolaterally, pushes the vessels of the carotid space medially or posteromedial and displaces the submandibular gland anteriorly. It may also be seen more medially within the parapharyngeal space after extending through the stylomandibular tunnel and middle constrictor muscle.
    MR imaging better depicts the deep tissue extent of a second branchial cleft cyst, which allows accurate preoperative planning. The cyst fluid varies from hypointense to slightly hyperintense relative to muscle on T1 weighted images and is usually hyperintense on T2-weighted images. Mural thickness and enhancement vary, depending of the presence and severity of any associated inflammatory process. Occasionally, a ‘beak sign’ may be seen on axial CT or MR images. This sign represents a curved rim of tissue or ‘beak’ pointing medially between the internal and external carotid arteries. It is considered a pathognomonic imaging feature of a second branchial cleft cyst.
How well did you know this?
1
Not at all
2
3
4
5
Perfectly
48
Q
  1. A neonate presents with an imperforate anus and a presacral mass. MRI of the abdomen demonstrates an enhancing lesion with fatty components and T2* susceptibility artefacts. What other imaging finding would you expect?
    A. Syrinx within the spinal canal
    B. Anterior beaking of the vertebral bodies
    C. Sacrococcygeal bony defect
    D. Developmental dysplasia of the hips
    E. Multiple renal cysts
A
  1. C. Sacrococcygeal bony defect
    Anterior sacral meningocele may occur as part of the Currarino triad, also known as the ASP triad (anorectal malformation, sacrococcygeal osseous defect and presacral mass), a rare syndrome characterised by autosomal dominant genetic inheritance in more than 50% of cases.
    The presacral mass in those affected may be a teratoma, anterior sacral meningocoeles, dermoid cyst, hamartoma or enteric duplication cyst, or more than one of these types of masses may be present.
How well did you know this?
1
Not at all
2
3
4
5
Perfectly
49
Q
  1. A 17-year-old boy presents with an enlarging swelling in the midline at the level of the hyoid bone. The GP suspects a thyroglossal cyst and sends him for an ultrasound. Which of the following appearances is consistent with a thyroglossal cyst?
    A. Hyperechoic mass in the midline
    B. Hyperechoic mass to the left of the midline
    C. Anechoic mass in the midline
    D. Hyperechoic mass with coarse internal echoes
    E. Hypoechoic mass to the left of midline
A
  1. C. Anechoic mass in the midline
    On all radiologic images, a thyroglossal duct cyst manifests as a cyst-like mass either in the midline of the anterior neck at the level of the hyoid bone or within the strap muscles just off the midline.
    At US, the finding of an anechoic mass with a thin outer wall in this characteristic location easily establishes the diagnosis of a thyroglossal duct cyst. However, this ‘classic’ appearance is seen in less than half (42%) of cases. More commonly, these cysts manifest as hypoechoic masses, often with increased through-transmission. They may be either homogeneous or heterogeneous in appearance with variable degrees of fine to coarse internal echoes. There is no correlation between the sonographic appearance and pathologic evidence of infection and inflammation. Heterogeneity seen in thyroglossal duct cysts on sonograms is more likely due to the proteinaceous content of the fluid secreted from the cyst wall rather than to infection. Preoperative sonographic visualisation of normal thyroid tissue is sufficient to exclude a diagnosis of ectopic thyroid tissue and obviates routine thyroid scintigraphy.
50
Q
  1. A 69-year-old woman presented to her GP with one month’s history of headache and progressive swelling of both hands and face and an 8 months’ history of progressive weight loss. Chest radiograph showed widening of the mediastinum and a large mass in the right upper lobe with a midline trachea. What do you expect the CT chest to show?
    A. Cavitating lung primary in RUL without mediastinal nodes
    B. Right Pancoast tumour with rib destruction
    C. RUL large and multiple small nodules of Wegeners granulomatosis
    D. RUL lung primary with retrosternal goitre
    E. RUL lung primary’ and SVC obstruction
A
  1. E. RUL lung primary and SVC obstruction
    Obstruction of the superior vena cava results in impaired venous drainage of the head and neck and upper extremities. Clinical manifestations include facial and neck swelling, distended neck veins, headache due to cerebral oedema, dyspnoea, stridor and altered mental status. Cancer is the most common underlying cause of superior vena cava obstruction and this includes lung cancer, mediastinal tumours, lymphoma/lymphadenopathy and mesothelioma, either directly or through malignant mediastinal lymphadenopathy. Other causes include catheter-induced iatrogenic SVC obstruction, fibrosing mediastinitis and Behcet’s disease.
51
Q
  1. A 63-year-old man presents to his general practitioner having noticed recent lower gastrointestinal bleeding. While waiting for his outpatient appointment he experiences massive lower gastrointestinal blood loss. The patient is unable to have a colonoscopy and is sent for an urgent angiogram. This demonstrates patency of the major mesenteric and coeliac vessels. An abnormal tuft of vessels with early filling of the accompanying mesenteric vein is noted at the caecum. No associated mass was seen. What is the most likely diagnosis?
    A. Ischaemic colitis
    B. Angiodysplasia
    C. Haemorrhoids
    D. Colon carcinoma
    E. Diverticulitis
A
  1. B. Angiodysplasia
    The clinical history and findings on angiogram suggest angiodysplasia. There is no history of atherosclerosis elsewhere to suggest ischaemic colitis. The major mesenteric vessels are usually patent in ischaemic colitis. The exception to this rule would be in a case of possible superior mesenteric artery obstruction. Angiodysplasia can be part of the hereditary haemorrhagic telangiectasia syndrome. The condition results from a chronic intermittent obstruction of the veins where they penetrate the circular muscle layer of bowel. Distorted dilated vascular channels replace the normal mucosal structures. Endovascular treatment is an option. The findings on angiography do not point to colon carcinoma, diverticulitis or haemorrhoids.
52
Q
  1. An ultrasound scan performed on a patient with spinal trauma demonstrates features of a neurogenic bladder. Which of the following features arc most commonly encountered?
    A. Thin walled bladder with decreased capacity
    B. Entirely normal bladder
    C. Thin-walled bladder with increased opacity
    D. Trabeculated bladder with increased opacity E. Trabeculated bladder with decreased capacity
A
  1. E. Trabeculated bladder with decreased capacity
    Neurogenic bladder can be caused by meningomyelocele, spinal trauma, diabetes, multiple sclerosis, polio and herpes infection. It is a heterogeneous condition: for example, it can cause incontinence owing to detrusor over-activity, difficulty in passing urine owing to detrusor underactivity or retention with overflow incontinence. Most commonly, the bladder eventually becomes trabeculated with decreased capacity, but very large atonic bladders can also be seen. The detrusor muscle is innervated by the sympathetic nerves originating from S2-S4, therefore, neurogenic bladder is a common problem in spinal trauma. Indwelling urethral or suprapubic catheters may be seen and can be associated with stone disease and infection, including pyelonephritis. Treatments that may be seen on imaging include urinary diversion and continent pouch formation.
53
Q
  1. A 40-year-old golfer presents with sudden-onset right medial palmar wrist pain during a golf swing. On examination, he has paraesthesia in the right fourth and fifth digits. Plain AP and oblique radiographs are normal. Which of the following is the most appropriate next investigation?
    A. Nerve conduction studies
    B. CT scan of the wrist with 1 mm slices
    C. Hook of hamate radiographic views
    D. MRI of the wrist
    E. Ultrasound scan of the wrist
A
  1. D. MRI of the wrist
    The patient’s presentation is a classic description of a hook of hamate fracture. These often occur during racquet, bat or club sports. A fracture of the hook of hamate can narrow Guyon’s canal, compressing the ulnar neurovascular bundle and causing distal neuropathy of vascular compromise. Conventional radiographs even with multiple dedicated views only have a sensitivity’ of 72% for diagnosing this carpal fracture. CT with multiplanar reformatting would be a reasonable next investigation; however, in view of this patient’s neurological signs of ulnar nerve compression, MRI is the imaging modality of choice. This gives better soft-tissue definition w’hile still having a sensitivity and specificity approaching 100% for diagnosing fractures.
54
Q
  1. All of the following statements regarding presacral mass lesions in children are true, except:
    A. Anterior sacral meningocoele is associated with Marfan syndrome.
    B. Rectal duplication cysts arc high signal on T1W images.
    C. Tailgut cysts have high mucin content.
    D. Dermoid cysts have areas of high T1W signal, suggesting fat.
    E. Rectal duplication cysts may mimic lymphatic malformation
A
  1. B. Rectal duplication cysts are high signal on T1W images.
    Anterior sacral meningocoele may be accompanied by other anomalies or syndromes, too, including uterine, renal and bladder malformations; Marfan syndrome; and Type 1 neurofibromatosis.
    Benign sacrococcygeal teratomas arc predominantly cystic; have attenuation similar to that of fluid on CT scans; and may include bone, fat and calcification. Cystic areas typically have the appearance of fluid on T1-weighted and T2 weighted MR images. Areas of fatty tissue demonstrate high signal intensity on T1 weighted images, whereas calcification and bone are depicted as areas of signal void. The coccyx is always involved, even in benign sacrococcygeal teratoma. Malignant teratomas have a predominant solid component, and haemorrhage and necrosis are common.
    Enteric cysts are rare and may arise in locations between the rectum and sacrum. They are classified according to their histologic basis as either tail-gut cysts or rectal duplication cysts. Duplication cysts on MR imaging demonstrates well-marginated, thin-walled lesions with low signal intensity on T1-weighted images and high signal intensity on T2-weighted images. Mucoid contents of tail gut cysts cause them to have high signal intensity on T1-weighted images.
    On MR images, cystic lymphatic malformations appear as areas of homogeneous high signal intensity on T2-weighted images and have the signal intensity of fluid on T1-weighted images.
    A fluid-fluid level can be seen in the setting of acute haemorrhage.
55
Q
  1. A 5-year-old girl with history of seizures and learning difficulty is referred for an abdominal ultrasound due to non specific abdominal pain. On imaging, you notice bilateral renal cysts and multiple hyperechoic lesions in the kidneys and spleen. The patient also has small red skin lesions on her face. What is the most likely unifying diagnosis?
    A. Sturge-Weber Syndrome
    B. Von Hippel-Lindau
    C. Autosomal dominant polycystic kidney disease
    D. Tuberous sclerosis
    E. PHAGE syndrome
A
  1. D. Tuberous sclerosis
    The US images suggest renal cysts, bilateral angiomyolipomas and splenic hamartoma. Polycystic kidney disease PCKD1 gene lies next to Tuberous sclerosis gene TSC2 and renal cysts are common in TS.
    TS is an autosomal, dominant, inherited neurocutaneous syndrome characterised by a variety of hamartomatous lesions in various organs. Classically, TS demonstrates a triad of clinical features (Vogt triad): mental retardation, epilepsy and adenoma sebaceum. PHACE syndrome constitutes, posterior fossa malformations-hemangiomas-arterial anomalies -cardiac defects-eye abnormalities-sternal cleft and supraumbilical raphe syndrome. Typically infants have large plaque like facial haemangiomas.
56
Q
  1. A patient with HIV is admitted to the medical ward with shortness of breath. Admission chest X-ray shows diffuse symmetrical air space change. There is no pleural effusion or mediastinal adenopathy. At what CD4 count would you expect Pneumocystis jiroveci to present as a potentially life threatening complication?
    A. CD4 <0
    B. CD4 <50
    C. CD4 <100
    D. CD4 <200
    E. CD4 <400
A
  1. D. CD4 <200
    Knowledge of the CD4 count when reporting a chest X-ray in an AIDS patient is important as opportunistic infections present at different count levels. A CD4 count lower than 200 is regarded as indicating a potentially life-threatening complication of Pneumocystis jiroveci infection.
    CD4 >500: No risk of opportunistic infection
    CD4 200-500: Candidiasis, Kaposi’s sarcoma
    CD4 100-200: P. jiroveci, histoplasmosis, coccidioidomycosis, PML
    CD4 50-100: Toxoplasma, cryptosporidiosis, cryptococcosis, CMV
    CD4 <50: Mycobacterium avium complex (MAC)
57
Q
  1. A 64-year-old man presents with left lower quadrant pain. He is very tender on clinical examination and an urgent CT scan with contrast is performed. The radiologist suggests that the findings are in keeping with epiploic appendagitis. Which of the following best describes the CT findings in epiploic appendagitis?
    A. Accordion sign
    B. Pericolic fat stranding
    C. Ascites
    D. Mucosal wall thickening with air-filled outpouchings
    E. Pericolic focal hyper attenuation with a central area of fat density
A
  1. E. Pericolic focal hyperattenuation with a central area of fat density
    Epiploic appendagitis is most common in the sigmoid colon or caecum, where the appendages arc most prominent. Most cases resolve spontaneously in about 2 weeks. It is therefore an important diagnosis to make, as it does not require surgery. It can appear on ultrasound as a non-compressible pericolic hyperechoic ovoid mass immediately under the abdominal wall. The accordion sign is seen in pseudomembranous colitis on post-contrast CT. Ascites is a non-specific sign that can be seen in a variety of conditions, including liver failure, trauma and pseudomembranous colitis. Mucosal wall thickening with air-filled outpouchings is characteristic of diverticulitis.
58
Q
  1. A 16-year old boy presents with recurrent urinary tract infections and an MRI Urogram is performed. ‘Phis confirms ureteric duplication. Which of the following is correct regarding ureteric duplication?
    A. The lower pole moiety inserts inferior to the upper pole moiety.
    B. The lower pole moiety is more likely to obstruct than the upper pole moiety.
    C. The upper pole moiety is often associated with an ectopic ureterocoele.
    D. The upper pole moiety inserts horizontally and is associated with reflux.
    E. Calyceal dilatation is usually seen in the lower pole
A
  1. C. The upper pole moiety is often associated with an ectopic ureterocoele.
    Ureteric duplication is seen in around 1%-2% of the population and is usually unilateral. The upper pole moiety inserts inferiorly and is often associated with an ectopic ureterocoele, which is more likely to obstruct, causing upper pole calyceal dilatation. The lower pole ureter inserts horizontally into the bladder, making it prone to vesicoureteric reflux, which can lead to scarring and deformity of the obstructed kidney’s lower pole. Intravenous urography may show poor or no function in the dilated obstructed upper pole. An MR or CT Urogram may also demonstrate the ‘drooping lily’ sign caused by the lower pole moiety being displaced inferiorly.
59
Q
  1. A 25-year-old woman had radiographs of her pelvis and both her lower limbs following a road traffic accident. Although no fracture was identified, multiple incidental findings were observed, including bilateral posterior iliac horns, protuberant anterior iliac spines and rudimentary patellae. Which of the following is the most likely diagnosis?
    A. Osgood-Schlatter
    B. Bipartite patella
    C. Diastrophic dysplasia
    D. Protrusio acetabuli
    E. Nail-patella syndrome
A
  1. E. Nail-patella syndrome
    Nail-patella syndrome is a rare autosomal dominant disorder characterised by symmetrical mesodermal and ectodermal abnormalities. Radiographic abnormalities include bilateral posterior iliac horns in 80% (diagnostic), flared iliac crests with protuberant anterior iliac spines, genu valgum owing to asymmetrical development of the femoral condyles, prominent tibial tubercles, fragmentation/hypoplasia/absence of the patella, radial head/capitellum subluxation with dislocation of the radial head dorsally, short fifth metacarpal, clinodactyly of the fifth finger, scoliosis and, occasionally, mandibular cysts. Protrusio acetabuli and bipartite patella are not disease entities as such; they are radiological descriptions of the acetabular floor bulging into the pelvis and a congenital fragmentation or synchondrosis of the patella, respectively. Diastrophic dysplasia refers to severe rhizomelic dwarfism, and Osgood-Schlatter refers to rupture of the growth plate at the tibial tuberosity secondary to an apophyseal traction injury.
60
Q
  1. A 60-year-old woman with breast carcinoma presents with headaches. CT shows erosion of the right foramen ovale on bone windows. An MRI of the head and neck region is performed. Which of these options is a likely finding?
    A. Fatty infiltration of the left side tongue muscles
    B. Fatty atrophy of the right masticator muscles
    C. Vocal cord deviation
    D. Swelling of the right side of the face
    E. Sensory loss of the right lower eyelid and cheek
A
  1. B. Fatty atrophy of the right masticator muscles
    Denervation and fatty atrophy of the pterygoid muscles occurs due to involvement of the motor portion of cranial nerve V supplying the muscle of mastication. This nerve runs through the foramen ovale.
61
Q
  1. A 5 month-old infant presents with one episode of urinary tract infection. The patient responds well to treatment and urinalysis reveals E. coli as the causative organism. There is no family history of ureteric reflux or renal disease. According to NICE guidelines, what imaging test(s) should be recommended?
    A. Urgent urinary tract ultrasound
    B. Routine urinary tract ultrasound within 6 weeks
    C. Routine urinary tract ultrasound with DMSA within 4-6 months
    D. Routine urinary tract ultrasound, DMSA and micturating cystourethrogram
    E. Magnetic resonance cystourethrogram
A
  1. B. Routine urinary tract ultrasound within 6 weeks

Atypical UTI includes the following:
Seriously ill, poor urine flow, abdominal or bladder mass, raised creatinine, septicaemia, failure to respond to treatment with suitable antibiotics within 48 hours, infection with non-E. coli organisms.
Recurrent UTI includes the following:
Two or more episodes of UTI with acute pyelonephritis/upper urinary tract infection, or one episode of UTI with acute pyelonephritis/upper urinary tract infection plus one or more episode of UTI with cystitis/lower urinary tract infection, or three or more episodes of UTI with cystitis/lower urinary tract infection.

Recommended Imaging Schedule for Infants Younger than 6 Months

62
Q
  1. A 55-year-old woman is found to have an incidental, pleurally based 7 cm ovoid mass with smooth tapered margins, on an otherwise unremarkable CT urography examination. MR imaging is performed for further assessment. Which of the following imaging characteristics is most suggestive of a diagnosis of benign mesothelioma?
    A. T1 MR hypointense; T2 MR hyperintense; contrast enhancement on CT avid
    B. T1 MR hypointense; T2 MR hyperintense; contrast enhancement on CT minor
    C. Tl MR hyperintense; T2 MR hyperintense; contrast enhancement on CT avid
    D. Tl MR hypointense; T2 MR hypointense; contrast enhancement on CT avid
    E. Tl MR hyperintense; T2 MR hyperintense; contrast enhancement on CT minor
A
  1. A. Tl MR hypointense; T2 MR hyperintense; contrast enhancement on CT avid Benign mesothelioma is otherwise known as solitary fibrous tumour of the pleura. Unlike
    malignant mesothelioma, it has no recognised association with asbestos exposure. It is usually solitary and more commonly arises from the visceral pleura. It is asymptomatic in 50% and is a recognised cause of finger clubbing, hypertrophic pulmonary arthropathy and episodic hypoglycaemia. It is usually sessile but may be pedunculated, in which case shape and location may vary with the patient’s position. Imaging characteristics are as above, and contrast enhancement may be heterogeneous if there is myxoid degeneration and haemorrhage. About 14%-30% undergo malignant degeneration. Surgical excision is curative.
63
Q
  1. A patient with known ulcerative colitis is admitted as an emergency following an episode of acute abdominal pain. The clinicians are worried about toxic megacolon and request an
    abdominal radiograph. Which of the following findings on the radiograph would be most specific for toxic megacolon?
    A. Preservation of haustra
    B. Thumbprinting of the mucosal wall
    C. Descending colon diameter of 4.5 cm
    D. Dilated transverse colon with mucosal islands
    E. Pseudo-diverticulae in the ascending colon
A
  1. D. Dilated transverse colon with mucosal islands
    Toxic megacolon and perforation are major complications that account for most ulcerative colitis-related deaths. Toxic megacolon is a fulminant form of colitis where inflammation becomes transmural and ulceration leads to neuromuscular degeneration and loss of muscle tone.
    347
    Recommended Imaging Schedule for Infants and Children between 6 Months and 3 Years
    Test
    Responds well to treatment within 48 hours
    Atypical
    UTI
    Recurrent
    UTI
    Ultrasound during the acute infection
    Ultrasound within 6 weeks DMSA 4-6 months following the acute infection MCUG
    No
    No
    No
    No
    Yes
    No
    Yes
    No
    No
    Yes
    Yes
    No
    Recommended Imaging Schedule for Children 3 Years or Older
    Test
    Responds well to treatment within 48 hours
    Atypical
    UTI
    Recurrent
    UTI
    Ultrasound during the acute infection
    Ultrasound within 6 weeks DMSA 4-6 months following the acute infection MCUG

The transverse colon is the least dependent part of the colon in the supine patient and therefore gas collects in this location. Dilatation occurs and perforation is likely. Radiographic findings include marked dilatation of the colon with absence of haustral markings, oedema and thickening of the colon wall, pneumatosis coli and perforation. Barium studies should not be performed because of the risk of perforation.

64
Q
  1. A 12-year-old boy undergoes ultrasound of his renal tract to investigate recurrent urinary tract infections. The kidneys initially appear to lie more inferiorly and anteriorly than would normally be expected. On closer examination, a bridge of renal tissue connects the lower poles of both kidneys and a diagnosis of horseshoe kidney is made. What structure has limited the ascent of the horseshoe kidney, causing its abnormal location?
    A. Umbilical ligament
    B. Coeliac axis
    C. Inferior mesenteric artery
    D. Superior mesenteric artery
    E. Attachment to the dome of the bladder
A
  1. C. Inferior mesenteric artery
    As the horseshoe kidney ascends, the isthmus comes in contact with the inferior mesenteric artery and halts it, causing the renal pelves to lie inferior and anterior to their normal position. Horseshoe kidney is the most common renal fusion abnormality, which is seen in 1-4 per 1,000 individuals. It occurs when the two kidneys are attached in the midline by their lower poles. There is an increased risk of calculi and infection because of urinary stasis. The risk of trauma is also increased, as in compressive injuries of the abdomen the midline isthmus of the horseshoe is compressed against the spine. In addition, as the kidney is lower-lying than normal, it is less protected by the thoracic cage. There is a three to four times greater risk of malignancies including transitional cell carcinoma, Wilms and occasionally carcinoid.
65
Q
  1. A plain film of the knee of a 33-year-old man involved in an RTA shows an pure depression fracture involving the lateral tibial plateau. What would be the Schatzker classification?
    A. Type I
    B. Type II
    C. Type III
    D. Type IV
    E. Type V
A
  1. C. Type III
    The Schatzker classification grades tihial plateau fractures in order of increasing severity based on the pattern of injury.
    Type I: Lateral tibial plateau split fracture, no depression
    Type II: Lateral tibial plateau split with depression; most common type
    Type III: Lateral tibial plateau pure depression only
    Type IV: Split and/or depressed fracture of medial tibial plateau
    Type V: Fracture of both medial and lateral tibial plateau with the metaphysis still in continuity with the diaphysis
    Type VI: Fracture of both tibial plateau and transverse subcondylar tibial fracture with separation of the metaphysis from the diaphysis
66
Q
  1. A 55-year-old postmenopausal woman is being treated with tamoxifen for breast cancer. Which of the following might occur as a side effect of this medication?
    A. Increased size of her uterine fibroids R. Reduction in size of her uterine fibroids
    C. Formation of multiple ovarian cysts
    D. Cervical stenosis
    F. . Thin atrophic endometrium
A
  1. A. Increased size of her uterine fibroids
    Tamoxifen has weak oestrogen agonist effects in the uterus despite working as an oestrogen antagonist in the breast, to treat breast cancers. Uterine fibroids are oestrogen sensitive and therefore instead of regressing postmenopausally, these women may experience an increase in symptoms because of their fibroids (as many women taking hormone replacement therapy). Tamoxifen can also cause endometrial hyperplasia and an increased risk of endometrial carcinoma.
    Formation of multiple ovarian cysts occurs in ovarian hyperstimulation syndrome as a result of clomifene therapy (taken for infertility) and in polycystic ovary syndrome where multiple cysts are characteristically seen around the periphery of enlarged ovaries.
67
Q
  1. A 33-year-old man with chronic sinus disease is being considered for a functional endoscopic sinus surgery (FESS) operation. The ENT surgeon refers him for CT sinuses prior to surgery. Regarding the ethmoidal sinuses, the ethmoidal air cells occasionally pneumatize laterally and posteriorly around the frontal recess. What is the name of this anatomical variation?
    A. Concha bullosa
    B. Agger nasi cell
    C. Haller cell
    D. Onodi cell
    E. Paradox middle turbinate
A
  1. B. Agger nasi cell
    Agger nasi is a Latin term literally meaning nasal mound’. At rhinoscopy, the agger nasi appears as an eminence located on the lateral nasal wall at the leading edge of the middle turbinate; it represents the intranasal portion of the frontal process of the maxilla. As noted earlier, the agger nasi serves as the anterior limit of the frontal recess. Pneumatisation of the agger nasi (resulting in the so-called agger nasi cell) occurs in 78%-98.5% of individuals.
    When present, agger nasi cells are considered the most anterior of all ethmoid cells and can pneumatise posteriorly to narrow the frontal recess. Coronal and sagittal reformatted CT images are most helpful in identifying the agger nasi cell. On coronal images, the agger nasi appears as a laterally placed sinus below the frontal sinus and anterior to the middle turbinate. Sagittal images demonstrate the anterior location of the air cell. Haller cells are extramural ethmoidal air cells extending to the inferomedial orbital floor. Onodi cells are posterior-most ethmoidal cells which lie superolateral to the sphenoid sinus in close relation to the optic nerve and internal carotid artery.
68
Q
  1. A 40-year-old man presents with dyspnoea and peripheral oedema and is found to have elevated jugular venous pressure. Echocardiography shows reduced mobility of the pericardium. Contrast-enhanced CT is performed. Which of the following features is least suggestive of a diagnosis of constrictive pericarditis?
    A. Pleural effusion
    B. Contrast reflux into the coronary sinus
    C. Curvature of the interventricular septum to the right
    D. Linear pericardial calcifications
    E. Azygos vein dilatation
A
  1. C. Curvature of the interventricular septum to the right
    Constrictive pericarditis is most common in men aged 30-50 years. It most commonly occurs following cardiac surgery although it is often idiopathic, with other aetiologies including viruses such as Coxsackie B, tuberculosis and uraemia. The computed tomography scan may show a normal-sized heart with superior vena cava, inferior vena cava and azygos vein dilatation. There is flattening of the right ventricle and curvature of the interventricular septum to the left (rather than to the right) because of high right-ventricular filling pressures. Pericardial thickening and calcification is a feature, as are pleural effusions. Treatment involves surgical stripping of the pericardium. If untreated, protein-losing enteropathy may occur as a result of raised inferior vena cava and portal venous pressures.
69
Q
  1. A 65 year old woman with known histoiy of gallstones is referred for an out-of-hours ultrasound scan to investigate fever, rigors and deranged liver function tests. Ultrasound demonstrates a common bile duct measuring 11 mm diameter and mild intrahepatic biliary dilatation, and a gallbladder containing multiple calculi associated with wall thickening, pericholecystic free fluid and a positive Murphy’s sign. What diagnosis should be fed back to the referring clinicians?
    A. Ascending cholangitis
    B. Biliary colic
    C. Acute cholecystitis
    D. Primary sclerosing cholangitis
    F. . Impacted calculus in common bile duct
A
  1. A. Ascending cholangitis
    The overwhelmingly important diagnosis not to be missed in this scenario is ascending cholangitis. This is because ascending cholangitis needs immediate treatment with broad-spectrum antibiotics and biliary drainage via either cholecystectomy or acute laparoscopic cholecystectomy. Although a sonographic-positive Murphy’s sign and pericholecystic free fluid suggest a diagnosis of acute cholecystitis, this should not distract the clinician from the presence of cholangitis, which is confirmed by the presence of fever and jaundice and is primarily a clinical diagnosis rather than a radiological one. Acute cholecystitis is not a clinical emergency and in this case is an associated finding that likely predates ascending cholangitis.
70
Q
  1. A 65-year-old woman presents to the orthopaedic clinic with left hip pain 3 years following a total hip joint replacement (THR). The plain radiograph shows focal radiolucencies around the prosthesis with smooth endosteal scalloping. The patient is systemically well with normal inflammatory markers. Presentation and radiographic appearances are characteristic of which of the following?
    A. Mechanical loosening
    B. Infection
    C. Histiocytic response
    D. Stress shielding
    E. Heterotopic ossification
A
  1. C. Histiocytic response
    Histiocytic response describes a macrophage reaction to any component of the arthroplasty (e.g. metal, cement, plastic spacer). The condition tends to occur within 1-5 years after surgery and the characteristic radiographic finding is smooth endosteal scalloping. There is an increased risk of fracture through the affected bone and therefore close follow-up is required. Computed tomography may show a soft tissue component and appearances can be aggressive.
    Stress shielding refers to areas of lucency often in the femur following THR secondary to alterations in bone loading. It predisposes the patient to both periprosthetic fractures and loosening and is more common in uncemented prostheses.
71
Q
  1. An X-ray of the mandible demonstrates a well-defined, unilocular cyst with corticated margins at the ramus. The patient is known to have a history of multiple skin lesions. What is the most likely diagnosis?
    A. Radicular cyst
    B. Residual cyst
    C. Cementoma
    D. Odontogenic keratocyst
    E. Ameloblastoma
A
  1. D. Odontogenic keratocyst
    Odontogenic keratocysts (OKC) are most commonly located in the body or ramus of the mandible. Most OKCs possess destructive potential, with a high recurrence rate after resection. OKCs develop from the dental lamina, which is found throughout the jaw and overlying alveolar mucosa and is lined by stratified keratinising squamous epithelium. Thus, the cysts can occur throughout periapical or primordial regions. Unlike follicular cysts, OKCs can expand cortical bone and erode the cortex. Fortunately, malignant transformation of these lesions is rare.
    The lesion is multiloculated, often with daughter cysts that extend to the surrounding bone.
    Multiple OKCs in a young patient should raise the possibility of basal cell nevus syndrome (Gorlin-Goltz syndrome). Associated findings with this autosomal dominant disorder include midface hypoplasia, frontal bossing and prognathism, mental retardation and calcification of the falx cerebri.
72
Q
  1. A tibia and fibula radiograph is performed on a 2-month-old baby for leg swelling. The radiologist notices fine periosteal reaction along the diaphysis of the tibia. Which one of the following statements is false regarding neonatal periosteal reaction?
    A. Physiological periosteal reaction may be a normal finding in children up to the age of 6 months.
    B. Commonly encountered sites of physiological periosteal reaction may include the humerus, tibia and femur.
    C. If physiological periosteal reaction is present, it is typically asymmetric.
    D. The initial presentation of non-accidental injury may include periosteal reaction of a long bone.
    E. The presence of marked periosteal reaction may suggest an underlying congenital cardiac disease.
A
  1. C. If physiological periosteal reaction is present, it is typically asymmetric.
    Periosteal bone formation may be due to either physiologic or pathologic causes.
    Physiological periosteal new bone formation is seen in up to a third of infants during the
    first few months of life. They are benign, usually symmetric and involve the long bones (femur, tibia and humerus). Radiographs demonstrate one or more dense lines of periosteal reaction along the diaphyses.
    Pathologic bone formation generally results from an adjacent inflammatory process or a hypoxic or toxic stimulus. Common causes of pathologic periosteal reaction in children include trauma to the underlying bone. However, other causes such as hypervitaminosis A, prostaglandin therapy, cortical hyperostosis (Caffey’s disease), hypertrophic osteoarthropathy (primary and secondary), osteomyelitis, leukaemia, trauma and syphilis must also be considered. The last four are usually associated with some degree of bone destruction, whereas in the first four diseases the underlying bone is left radiologically intact.
73
Q
  1. A 16-year-old boy presents with shin pain. A tibia and fibula radiograph demonstrates a well-defined, cortically based osteolytic lesion within the diaphysis measuring 4 cm cranio-caudally without periosteal reaction.
    On MRI, the lesion is solitary’, intermediate signal on T1 sequences (same as muscle) and high signal on T2. There is homogenous and avid enhancement post-contrast administration. There is no soft-tissue mass associated with the bony lesions.
    Which one of the following differential diagnoses is the most likely?
    A. Simple bone cyst
    B. Non-ossifying fibroma
    C. Fibrous cortical defect
    D. Chondroid myxoid fibroma
    E. Adamantinoma
A
  1. E. Adamantinoma
    Adamantinoma appears as a well-circumscribed, slightly expansile lesion, usually with a narrow zone of transition, a finding consistent with its indolent nature. It is often multilocular, with sclerosis and lysis seen in a ‘soap bubble’ pattern. The lesion is typically oriented longitudinally along the anterior tibia! diaphysis, with an average length of 10 cm.
    MR imaging is the best modality for delineating the extent of adamantinoma in the medullary cavity, soft tissues and satellite lesions. These tumours typically demonstrate intermediate signal intensity relative to muscle on T1-weighted images and intensity similar to that of fat on T2-weighted images obtained without fat saturation. Static-enhanced images demonstrate intense homogeneous enhancement within the lesion.
74
Q
  1. A 73-year-old man is found to have a pulmonary nodule on routine chest radiograph.
    Pre- and post-contrast-enhanced CT of the chest is performed. Which one of the following features is least suggestive of benignity of the nodule?
    A. Nodule enhancement of 10 Hounsfield units
    B. A peripheral rim of enhancement
    C. A flattened configuration of the nodule
    D. Central calcification
    E. Ground glass and solid components to the nodule
A
  1. E. Ground-glass and solid components to the nodule
    A variety of features of solitary pulmonary nod ides suggest a benign aetiology. If computed tomography scans have been performed pre and post-contrast administration, assessment of the degree of enhancement can be performed. Enhancement of less than 15 Hounsfield units strongly
    favours benignity, whereas enhancement of greater than 15 Hounsfield units could represent malignancy or an inflammatory process. A peripheral rim of enhancement implies benignity but can occur with cavitating tumours such as squamous cell carcinoma. A flattened nodule (axial: craniocaudal diameter ratio of >1.78:1) is also indicative of a benign aetiology, as is central calcification. Partly solid nodules (i.e. solid and ground glass components) are more likely to represent malignancy, in particular bronchioloalveolar cell carcinoma.
75
Q
  1. A 24-year-old man is investigated for weight loss and chronic abdominal pain. Small bowel MRI demonstrates thickened loops of small bowel. Small bowel follow-through confirms the presence of non-stenotic ulcers of the distal small bowel. Apart from Crohn’s disease, which one of the following is the most common diagnosis?
    A. Yersinia
    B. Tuberculosis
    C. Reiter syndrome
    D. Lymphoma
    E. Steroids
A
  1. A. Yersinia
    Yersinia enterocolitica is the most common cause of non-stenotic ulcers of the small bowel. It is a Gram-negative organism that causes fever, right iliac fossa pain and diarrhoea. It most commonly affects the terminal ileum. As a human pathogen, Y. enterocolitica is most frequently associated with acute diarrhoea, terminal ileitis, mesenteric lymphadenitis and pseudoappendicitis. It may be complicated by abscess formation and has been associated with reactive arthritis. Tuberculosis and lymphoma are also differential diagnoses; however they arc less common than yersinial infection. Occasionally, they can only be differentiated by direct visualisation and biopsy rather than by imaging characteristics.
76
Q
  1. A 30-year-old woman with dysmenorrhoea and menorrhagia has had uterine artery embolisation of her fibroid uterus. She presents with lower abdominal pain and a malodorous vaginal discharge. Ultrasound demonstrates a distended debris- and fluid-filled uterine cavity with a separate hypoechoic large mass noted at the internal cervical os, causing cervical obstruction. Which of the following is the most likely explanation for these findings?
    A. Embolisation and subsequent swelling surrounding a cervical fibroid
    B. Detachment of a large subserosal pedunculated fibroid following embolisation
    C. Detachment of a large submucosal fibroid following embolisation
    D. Incidental large endometrial polyp close to the internal cervical os
    E. Cervical carcinoma
A
  1. C. Detachment of a large submucosal fibroid following embolisation
    Uterine artery embolisation is an increasingly popular treatment that is used for the treatment of troublesome uterine fibroids; however, complications have been reported. Fibroid passage is one such complication; this is defined as detachment of the treated uterine fibroid. An increased risk of fibroid passage is seen in fibroids abutting the endometrial cavity (e.g. submucosal fibroids). Fibroid passage can precipitate severe pain, infection or recurrent bleeding. In this case, fibroid passage has caused cervical obstruction and pyometra. Cervical carcinoma and a large endometrial polyp can both cause cervical stenosis and consequent pyometra, but they are both much less likely in this circumstance.
77
Q
  1. A 20-year-old man was seen in the A&E department after an injury to his foot during a football game. On examination, there was tenderness on palpation in the forefoot. A plain film performed showed a step in the alignment of the medial aspect of the second metatarsal bone and middle cuneiform bone. The second to fourth metatarsal had moved laterally. Normal alignment was noted at the articulation between the first metatarsal and medial cuneiform bone. Which type of fracture is demonstrated on the plain film?
    A. March fracture
    B. Jones fracture
    C. Lover’s fracture
    D. Homolateral Lisfranc fracture dislocation
    E. Divergent Lisfranc fracture dislocation
A
  1. D. Homolateral Lisfranc fracture dislocation
    Lisfranc injury is a fracture dislocation of the tarsometatarsal joints in a dorsolateral direction. The second metatarsal is held in a mortise formed by the medial and lateral cuneiform bones. Transverse metatarsal ligaments, both on the plantar and dorsal surfaces, connect the second to fifth metatarsals proximally. There is no transverse ligament between the first and second metatarsal. The plantar ligaments are stronger; hence most dislocations occur dorsally. If there is medial dislocation of the first metatarsal and lateral dislocation of the remaining metatarsals, it is considered a divergent type of Lisfranc injury. If the alignment of the first metatarsal is normal, or if it displaces laterally along with the remaining metatarsals, it is considered the homolateral type. On AP radiography of the foot, the medial aspect of the second metatarsal and middle cuneiform bone should align and, on the oblique view, the medial aspect of the third metatarsal and the lateral cuneiform bone should align.
    March fracture is another name for stress fracture, which commonly occurs at the distal shafts of the second and third metatarsals. They may appear on plain radiograph as a lucent line or their presence may be noted secondary’ to faint periosteal reaction or callus formation. If not visible on plain films and clinical suspicion persists, MR] or radionuclide scan may be helpful.
    Lover’s fracture is another term for fracture of the calcaneus. Jones fracture occurs in the proximal shaft of the fifth metatarsal. It has a high rate of non-union and is found within 1.5 cm of the fifth metatarsal tuberosity.
78
Q
  1. A 20-year-old patient presents with long-standing halitosis and foul taste in the
    mouth. MRI showed a 1.5 cm smoothly marginated midline cystic mass in the posterior roof of the nasopharynx, which does not enhance with contrast. What is the likely diagnosis?
    A. Fossa of Rosenmüller tumour
    B. Rathke’s pouch
    C. Thornwaldt’s cyst
    D. Mucous retention cyst
    E. Chronic sinus disease
A
  1. C. Tornwaldt’s cyst
    Tornwaldt’s cyst or pharyngeal bursa is an epithelial cyst that is related to the embryogenesis of the notochord. The incidence of Tornwaldt’s cysts has been reported in 4% of autopsy specimens and from 0.2% to 5% on MR observation. They occur in the posterior wall of the nasopharynx.
    Although Tornwaldt’s cysts are present in all age groups, the peak incidences have been reported to occur in various age groups.
    The three major symptoms are persistent, noticeable nasal discharge; obstinate occipital headaches; and halitosis and an unpleasant taste in the mouth. According to researchers who have used MR imaging to study Tornwaldt’s cyst, this lesion has characteristic high signal intensity on T1-weighted and T2-weighted images.
79
Q
  1. ‘Floating tooth’ sign is not associated with which one of the following conditions?
    A. Leukaemia
    B. Multiple myeloma
    C. Multifocal histiocytosis
    D. Burkitt lymphoma
    E. Mastocytosis
A
  1. E. Mastocytosis
    Multifocal histiocytosis has a triad (only 10% of patients) of calvarial lesion, exophthalmos and diabetes insipidus. About 40% of patients with Hand-Schüller Christian disease (multifocal histiocytosis) have destructive lesions affecting the alveolar margins on the mandible. The surrounding radiolucent matrix gives the appearance of floating teeth.
    Other conditions such as leukaemia, multiple myeloma and Burkitt lymphoma can also give the appearance of floating teeth. Mastocytosis can cause focal or diffuse sclerotic lesions and does not give the appearance of floating teeth.
80
Q
  1. A 40-year old man is diagnosed on the basis of a screening chest radiograph and a subsequent high-resolution thoracic CT scan with alveolar microlithiasis. Which one of the following is not an expected finding in this condition?
    A. Raised serum calcium levels
    B. Diffuse involvement of both lungs
    C. Intense uptake on bone scintigraphy
    D. Submillimeter micronodulations
    E. Marked discrepancy between radiograph and clinical symptoms
A
  1. A. Raised serum calcium levels
    The aetiology of alveolar microlithiasis is uncertain, and the condition is characterised by multiple tiny intra-alveolar opacities known as calcospherites. The peak age of those affected is 30-50 years, although onset is at a young age, with antenatal onset having been reported. On biochemical profiling, serum calcium and phosphate levels are both normal. There is diffuse involvement of both lungs by submillimetre micronodulations. Uptake on bone scintigraphy is intense. The majority of patients are asymptomatic, although shortness of breath on exertion, cyanosis and digital clubbing may be present. Lungs may appear more dense than the heart shadow on a chest radiograph, and radiographic features are disproportionate to the degree of symptoms.
81
Q
  1. A 42-year-old patient who has a long history of ileocolic Crohn’s disease presents with weight loss and vomiting. On CT, there is a heterogeneous soft-tissue polypoidal mass halfway along the second part of the duodenum. No significant lymphadenopathy is noted. Which of the following is the most likely diagnosis?
    A. Adenocarcinoma of the small bowel
    B. Hamartomatous polyp
    C. Lymphoma
    D. Gastrointestinal stromal tumour (GIST)
    E. Pancreatic carcinoma
A
  1. A. Adenocarcinoma of the small bowel
    Crohn’s disease is a risk factor for the development of small bowel tumours, including lymphoma and small bowel adenocarcinoma. Lymphoma, however, is usually associated with lymphadenopathy and rarely involves the duodenum, being more common in the ileum or stomach. It is also more commonly extraluminal. In contrast, adenocarcinoma is most commonly intraluminal. Hamartomatous polyps arc benign polyps usually associated with Peutz-Jeghers syndrome, which is also a risk factor for small bowel adenocarcinoma. Gastrointestinal stromal tumours are usually extramural exophytic masses. Pancreatic carcinoma is not usually associated with Crohn’s disease but could cause a bulky tumour mass in this region, although it should arise from the pancreas and invade the duodenum.
82
Q
  1. Routine mammograms of a woman show a 1 cm retro-areolar lesion. Ultrasound demonstrates an 8 mm retro-areolar lesion with a dilated duct. Which of the following is the most likely diagnosis?
    A. Papillomatosis
    B. Ductal cancer
    C. Ductal ectasia
    D. Plasma cell mastitis
    E. Papilloma
A
  1. E. Papilloma
    Papillomas are benign lesions (hyperplastic epithelium on a stalk) usually found in an ectatic subareolar major duct. They are the most common cause of blood or serous discharge. Papillomatosis refers to multiple peripheral papillomas that are located in the duct lumen just proximal to the lobule; it carries an increased risk of malignancy. Ductal ectasia is a benign entity that represents accumulation of cellular debris in enlarged subareolar ducts. Presentation is usually with non-bloody discharge and/or pain. It is the second most common cause of blood or serous discharge. Plasma cell mastitis refers to an inflammatory component associated with extensive secretory calcification.
83
Q
  1. A 57-year-old woman presents with a dry mouth and dry eyes. Ultrasound of the parotids demonstrates multiple small hypoechoic areas. MRI demonstrates a speckled honeycomb appearance on T2W images. What is the most likely diagnosis?
    A. Sarcoid
    B. Pleomorphic adenoma
    C. Sjögren’s syndrome
    D. Warthin’s tumour
    E. Mucoepidermoid carcinoma
A
  1. C. Sjögren’s syndrome
    Most high grade mucoepidermoid carcinomas, undifferentiated carcinomas, adenocarcinomas and squamous cell carcinomas of the major salivary glands have low to intermediate signal intensity on long repetition time images. Some malignancies, however, exhibit elevated signal intensity on T2-weighted images. Most commonly, this is seen in low-grade mucoepidermoid carcinomas, in some adenoid cystic carcinoma, and rarely in adenocarcinomas. Among benign masses that are not hyperintense on long repetition time images, Warthin’s tumour, the second most common benign mass in the adult parotid gland, is often of intermediate, low or mixed signal intensity on T2-weighted images.
    The chronic sialadenitis, such as Sjögren disease, Mikulicz disease (also known as Sjögren Type 1 or sicca syndrome without a connective tissue disorder) and radiation sialadenitis may also appear hypointense on T2 weighted images. In most series, these exceptions account for the 25% error rate that occurs if one relies solely on signal intensity on T2-weighted images to predict histologic diagnosis.
84
Q
  1. Anteroposterior radiograph of the knee of a 14 year old boy shows proximal tibial physeal widening and irregularity. Because of continued knee pain worsening with activity, MR imaging is performed to exclude a meniscal tear. MRI shows substantial widening and irregularity of the proximal tibial physis. Physeal stress injury is diagnosed. All of the following are expected findings, except
    A. Cartilage injury
    B. Tongue of high signal extending into metaphysis
    C. Bone bridging
    D. Physeal irregularity
    E. Diffuse low T1W signal to the bone adjacent to the physis
A
  1. A. Cartilage injury
    Conventional radiographs demonstrate physeal widening, irregularity and fragmentation.
    MR imaging can be used to further characterise the injury by demonstrating oedema in the adjacent bone on T2-weighted or STIR MR images and focal extensions of unmineralised cartilage into the metaphysis, which appear as ‘tongues’ of high signal intensity on T2-weighted, STIR and gradient-recalled echo (GRE) images and as intermediate signal intensity on T1-weighted images. Continued overuse leads to further physeal widening, irregularity and occasionally bone bridging.
85
Q
  1. A 50-year-old man with chronic middle car problems has a CT scan of the temporal bones, which shows a cholesteatoma. Regarding cholesteatoma, which of the following statements is false?
    A. Cholesteatomas can be classified as either congenital or acquired.
    B. Congenital cholesteatomas can be located everywhere in the temporal bone.
    C. Primary acquired cholesteatomas develop behind an apparently intact tympanic membrane.
    D. Pars flaccida (attic) cholesteatomas are located at the lower one-third portion of the tympanic membrane.
    E. Pars tensa (sinus) cholesteatomas develop most often through a defect of the lower two-thirds portion of the tympanic membrane
A
  1. D. Pars flaccida (attic) cholesteatomas are located at the lower one-third portion of the
    tympanic membrane.
    Cholesteatomas can be classified as either congenital or acquired, though the origins are indistinguishable with histology and imaging. Only the location of the lesion, the clinical history of the patient, and the otologic status of the tympanic membrane (TM) give some idea how to differentiate these two types of cholesteatomas.
    Congenital cholesteatomas develop from embryonic epithelial rests and can be located anywhere in the temporal bone: in the middle ear, in the mastoid, in the petrous apex in the squama of the temporal bone, within the TM or in the external auditary canal (EAC). Furthermore, the same histologic entity can arise in other areas of the skull, in the extracranial soft tissues or in an intracranial extra-axial location, where it is referred to as epidermoid cyst. Middle ear congenital cholesteatomas represent approximately 2% of all middle ear cholesteatomas. Primary acquired cholesteatomas (80% of all middle ear cholesteatomas) develop behind an apparently intact TM, usually in the region of the pars flaccida.

Secondary acquired cholesteatomas (18% of all middle ear cholesteatomas) grow into the middle ear through a perforated TM, usually through the pars tensa and sometimes the pars flaccida. Pars flaccida (attic) cholesteatomas are located at the upper one third portion of the TM.
Pars tensa (sinus) cholesteatomas develop most often through a defect of the lower two-thirds portion of the TM.

86
Q
  1. A 45-vear-old woman is diagnosed with a biopsy-proven cancer. A lymph node is also demonstrated in the upper outer quadrant of the breast. Which of the following features is most suggestive of malignancy in the node?
    A. Hyperechoic centre
    B. Round and hypoechoic node
    C. Posterior acoustic shadowing
    D. A 1.5 cm node in the upper outer quadrant
    E. Homogeneous enhancement on MRI
A
  1. B. Round and hypoechoic node

US criteria for benign versus malignant nodes:

87
Q
  1. A 56-year-old man presents with dyspnoea. Chest radiography shows increased interstitial markings with a predominantly lower zone distribution bilaterally. A diagnosis of interstitial fibrosis is made on high-resolution CT. Which of the following conditions is least likely to show interstitial fibrosis with this distribution?
    A. Systemic lupus erythematosus
    B. Rheumatoid arthritis
    C. Ankylosing spondylitis
    D. Methotrexate-induced fibrosis
    E. Scleroderma
A
  1. C. Ankylosing spondylitis
    Ventilation in the lungs is better in the upper zones than in the lower zones. By contrast, perfusion is better in the lower zones than in the upper zones. Accordingly, there is a tendency for occupational dust exposure to cause interstitial fibrosis with a predominantly upper zone distribution (although asbestosis is a notable exception, with a lower zone distribution). Ankylosing spondylitis also has an upper zone distribution. Lower zone fibrosis tends to arise as a result of haematogenous factors. Hence, drug-induced fibrosis (e.g. that caused by methotrexate) and collagen vascular diseases (such as systemic lupus erythematosus, rheumatoid arthritis and scleroderma) show’ lower zone predominance. Idiopathic pulmonary fibrosis is the most common cause of lower zone interstitial fibrosis.
88
Q
  1. A 60-year old man with neurofibromatosis Type 1 presents with vague abdominal pain and bloating. A CT scan reveals a well-defined hyperenhancing rounded exophytic mass within the left upper quadrant, which has a broad base with the body of the stomach with no fat plane between the two. The spleen, transverse colon and left kidney are displaced by this mass. It measures
    18 cm. There is no lymphadenopathy. Which of the following is the most likely diagnosis?
    A. Ectopic pancreas
    B. Gastric lipoma
    C. Gastric adenocarcinoma
    D. Malignant GIST (gastrointestinal stromal tumour)
    E. Benign GIST (gastrointestinal stromal tumour)
A
  1. D. Malignant GIST (gastrointestinal stromal tumour)
    Gastrointestinal stromal tumours are the most common mesenchymal tumours of the gastrointestinal tract. They are usually found around the stomach and are often exophytic in nature. Commonly, they are found incidentally and can become very large before causing symptoms. These are difficult to distinguish from leiomyomas, leiomyosarcomas, schwannomas and neurofibromas radiologically and are characterised on immunohistochemistry by their
    expression of the KIT gene. It is important to appropriately identify GISTs as these can be treated successfully with tyrosine kinase inhibitors (Glivec®). Gastrointestinal stromal tumours can metastasise to the liver and peritoneum but lymphadenopathy is very uncommon. Once these tumours are over 5 cm malignancy becomes more likely.
    Lymphoma would usually be associated with lymphadenopathy. A gastric adenocarcinoma of this size would also be likely to have associated lymphadenopathy. Gastric lipoma should not enhance and be of low attenuation. Ectopic pancreas is usually endoluminal with an ill-defined border and is more often found in the antrum of the stomach and duodenum.
89
Q
  1. With regard to ossification centres of the elbow in a child, which of the following statements is correct?
    A. The absence of the internal epicondyle ossification centre suggests an avulsion injury, if the radial ossification centre is visible.
    B. The absence of the lateral epicondyle ossification centre could be normal in a 12-year-old.
    C. The trochlear ossification centre occurs before the capitellum ossification centre.
    D. Lateral epicondyle avulsion injuries are more common that medial epicondyle avulsion injuries.
    E. Medial epicondyle avulsion injuries are associated with excess forces from the common extensor tendon.
A
  1. B. The absence of the lateral epicondyle ossification centre is often a normal finding in a
    12-year old.
    The ossification centres occur in order of appearance according to the commonly recited mnemonic CRITOE - capitellum, radial, internal (medial epicondyle), trochlear, olecranon and external (lateral epicondyle). Although the associated age groups are commonly stated as 1, 3, 5, 7, 9 and 11 years of age, respectively, there is actually some variability, particularly as the final ossification centres appear. As such, the lateral epicondyle ossification centre may not appear until age 13. Medial epicondyle avulsion injuries are far more common than lateral epicondyle avulsion injuries. The common extensor tendon inserts onto the lateral epicondyle and the common flexor tendons insert onto the medial epicondyle.
90
Q
  1. With regard to physeal stress injury in children, all of the following statements are true, except;
    A. Little Leaguer’s shoulder refers to injury of the proximal humerus.
    B. Physeal injuries are more easily identified on MRI.
    C. Gymnast’s wrist refers to injury to the distal radius.
    D. Gymnast’s wrist results in negative ulnar variance.
    E. Little Leaguers elbow refers to injury to the medial epicondyle of the humerus.
A
  1. D. Gymnast’s wrist results in negative ulnar variance
    In Little Leaguer’s shoulder, excessive overhead throwing leads to widening and irregularity of the proximal humeral physis on radiographs and MR images. In gymnast’s wrist, repetitive weight-bearing on the wrist can lead to physeal stress changes of the distal portion of the radius and, occasionally, the ulna. Abnormal distal radial growth produces positive ulnar variance. At a later stage, this could be associated with injury to the TFC complex. The valgus stress of the cocking and acceleration phases of pitching results in a traction injury to the medial epicondyle that is known as Little Leaguer’s elbow. MR imaging is more sensitive than conventional radiography for detecting physeal changes.
91
Q
  1. A 34-year-old man is undergoing preoperative assessment of his mitral valve using CT angiography. Which of the following combinations represents the optimal view and timing for visualising the mitral valve on CT?
    A. Two-chamber view in mid-systole
    B. Two-chamber view in mid-diastole
    C. Three-chamber view in mid-systole
    D. Three-chamber view in mid-diastole
    E. Four-chamber view’ in mid-diastole
A
  1. B. Two-chamber view in mid-diastole
    In cardiac CT imaging, standard views include the four-chamber (both atria and ventricles), three-chamber (left ventricle and atrium and aortic root), two-chamber (left atrium and ventricle) and short axis projection views. Five-chamber views (both atria and ventricles and aortic root) can also be used. These views are usually formulated using maximal intensity projection technique on source images, with two-chamber, three-chamber and four-chamber view’s obtained by rotating the imaging plane along the left ventricle’s long axis. Combinations A through E will all demonstrate the mitral valve, but the two-chamber view taken in mid-diastolic is the optimal view and timing for the assessment of the mitral valve.
92
Q
  1. A 43-year-old diabetic man underwent non-contrast CT of his abdomen and pelvis, which demonstrated a dense liver. He described erectile dysfunction and arthropathy mainly affecting his second and third metacarpophalangeal joints bilaterally but also affecting his hips. Which of the following is the most likely diagnosis?
    A. Amyloidosis
    B. Glycogen storage disease
    C. Wilson disease
    D. Haemophilia
    E. Haemochromatosis
A
  1. E. Haemochromatosis
    The patient is likely to have undetected haemochromatosis; the dense liver will be caused by iron deposition and this would give a low signal on T2-weighted (especially T2*) magnetic resonance images. Deposition in the pancreas and gonads can cause diabetes, erectile dysfunction and hypogonadism, respectively. Haemochromatosis is often termed bronze diabetes, as there is hyperpigmentation in the skin because of the deposition of iron. It is known to cause osteoarthritic changes at many joints because of iron deposition, and the most characteristic place for this to occur is the metacarpo-phalangeal joints, especially the second and third, where there may also be hook-like osteophytes.
    Amyloidosis causes a decrease in the liver density. Wilson disease can give a dense liver and is associated with arthropathy, although this is usually of the large joints and spine; it does not cause diabetes and more often causes neuropsychiatric complaints. Haemophilia would tend to present as an arthropathy first involving multiple joints and at a younger age. The dense liver may be caused by repeated blood transfusions but is less likely to be associated with diabetes in this case. Gaucher disease (the most common glycogen storage disease) tends to present earlier in life, although late cases are not unheard of. They tend to present with splenomegaly, lytic bone lesions and haematological upset.
93
Q
  1. A 50 year-old woman presents with refractory hypertension and palpitations. Urinary vanillylmandelic acid (VMA) levels are raised. A CT scan demonstrates normal adrenals. Which of the following should be the next line of investigation?
    A. Ultrasound (US) scan of adrenal glands.
    B. In-phase and out-of-phase MRI sequences to assess fatty content.
    C. Radionuclide imaging using an analogue of guanethidine such as metaiodobenzylguanidine (MIBG).
    D. Adrenal biopsy.
    E. Radionuclide imaging using radiolabelled analogues of cholesterol such as 131I-6β- iodomethyl-19-norcholesterol (NP-59).
A
  1. C. Radionuclide imaging using an analogue of guanethidine such as
    metaiodobenzylguanidine (MIBG)
    An MIBG scan will help evaluate an extra-adrenal paraganglioma. Radionuclide imaging using an analogue of guanethidine such as MIBG is concentrated in sympatho-adrenal tissue and is thus used to image adrenal medullary disorders.
94
Q
  1. A 65-year-old male patient presents with back pain and loss of sensation in his left leg. An MRI scan of his lumbar spine reveals multiple lesions within the vertebral bodies of his lower spine, of varying sizes. These were of low signal on both T1 and T2 sequences. Which of the following is the most likely diagnosis?
    A. Myeloma
    B. Paget disease
    C. Prostate cancer metastases
    D. Renal cancer metastases
    E. Haemangiomas
A
  1. C. Prostate cancer metastases
    Metastases to bone are 15 100 times more common than primary skeletal neoplasms and are found at sites of dominant haematopoietic marrows because of its rich haematopoietic marrow. In adults, this includes the calvarium, spine, flat bones and proximal humeral and femoral metaphyses. Focal sclerotic lesions are hypointense on T1 and T2-weighted sequences. The most common metastases in adult men are due to prostate carcinoma. Renal metastases are usually lytic. Haemangiomas are hyperintense on both T1 and T2-weighted sequences
95
Q
  1. A 40-year-old woman with long-standing history of epistaxis has been diagnosed with Wegener’s granulomatosis. The following statements are true about Wegener’s granulomatosis affecting the head and neck except:
    A. It may lead to destruction of the hard palate.
    B. Premaxillary and retroantral fat are rarely involved.
    C. Sarcoidosis may have a similar appearance.
    D. Chronic disease may result in thickening of the paranasal sinuses.
    E. The nasal septum may disappear in chronic disease.
A
  1. B. Premaxillary and retro-antral fat are rarely involved.
    The combination of necrotising granulomatous lesions of the upper and lower respiratory tracts, generalised necrotising vasculitis of arteries and veins and glomerulonephritis is called Granulomatosis and polyangitis (previously called Wegener’s granulomatosis). The disease may affect the eyes, skin, joints, muscles and cardiac system as well as the paranasal sinuses, lungs and kidneys.
    Advanced Wegener’s granulomatosis may lead to destructive lesions of the hard palate, sinonasal-oral fistulas or complete nasal septal destruction.
    The premaxillary soft tissues, retro-antral fat, pterygopalatine fossa, infratemporal fossa and orbits are commonly involved. Sarcoidosis may have a similar appearance.
    When a destructive mass in the nasal septum extends into the orbits, Wegener’s granulomatosis should be considered in the differential diagnosis. Intracranial involvement may also result, usually in the anterior cranial fossa, owing to the spread of the disease through the cribriform plate.
    As the disease becomes chronic, the walls of the residual paranasal sinuses (particularly the maxillary sinuses) become markedly thickened while the sinus volume is gradually reduced, and the nasal septum may completely disappear.
96
Q
  1. A large posterior fossa mass arising from the roof of the fourth ventricle is identified on an MRI brain in an 8-year-old child. The lesion is poorly defined and does not contain any internal calcification or extension into the basal cisterns.
    Which imaging feature of this lesion would be considered atypical?
    A. Marked associated hydrocephalus
    B. Avid contrast enhancement
    C. Hyperdense appearance on CT
    D. Absence of drop metastases on MRI spine imaging
    E. Restricted diffusion on diffusion-weighted imaging (DWI)
A
  1. D. Absence of drop metastases on MRI spine imaging
    The description would fit with a medulloblastoma. Drop metastases arc commonly found at presentation, and complete imaging of the neuraxis is strongly recommended at diagnosis.
    Cerebrospinal fluid (CSF) seeding, with the resultant formation of spinal intradural ‘drop metastases’, is a well-known mode of dissemination for many intracranial neoplasms. This manner of tumour spread is most commonly observed in medulloblastoma. Ependymomas, anaplastic gliomas, germinomas, choroid plexus tumours and pineal parenchymal tumours (pineoblastomas and pineocytomas) are other intracranial tumours that frequently deposit metastases in the spinal canal.
    Intradural metastases may also occur as a result of haematogenous spread from extracranial neoplasms, most commonly lung and breast adenocarcinoma and from haematological malignancies.
97
Q
  1. A preterm neonate born at 24 weeks is referred for a cranial ultrasound study. A focus of increased echogenicity is present in the right caudothalamic groove extending into the lateral ventricle, without ventricular dilatation.
    Which grade germinal matrix haemorrhage would this feature most represent?
    A. Grade 1
    B. Grade 2
    C. Grade 3
    D. Grade 4
    E. Grade 5
A
  1. B. Grade 2
    The most common location for injury to the premature brain is the periventricular white matter, with ischaemic parenchyma manifesting as periventricular leukomalacia (PVL). Initial sonograms show hyperechogenic globular change in the periventricular regions, and MR images depict areas of T1 hyperintensity within larger areas of T2 hyperintensity. Subsequent cavitation and periventricular cyst formation, features that are required for a definitive diagnosis of PVL, develop 2-6 weeks after injury and are easily seen on sonograms as localised anechoic or hypoechoic lesions. The progressive change ventricular dilatation occurs, described as end-stage PVL Subsequent reperfusion to the ischaemic tissues in the setting of weakened capillaries and increased venous pressure result in germinal matrix haemorrhage, ranging in severity from subependymal haemorrhage (Grade 1) to intraventricular haemorrhage without (Grade 2) and with (Grade 3) ventricular dilatation, to parenchymal extension and coexisting periventricular venous infarction (Grade 4).
98
Q
  1. A 50 year old teacher with a neck lump is referred for an ultrasound, which revealed a thyroid nodule. Which of the following is an indication for FNA of a thyroid nodule?
    A. Entirely cystic lesion
    B. Presence of microcalcification
    C. Presence of multiple cystic nodules
    D. Every nodule above 2 cm
    E. Every nodule that is hypervascular
A
  1. B. Presence of microcalcification
    These are general recommendations for adult patients who have a thyroid nodule on US images, regardless of how the nodule was initially detected.
    The recommendations may not apply to all patients, including those who have historical, physical or any other features suggesting they arc at increased risk for cancer or who have a history of thyroid cancer.
    Part I
    The following are general recommendations for nodules 1.0 cm or greater in largest diameter. Solitary nodule:
    Strongly consider FNA for (1) a nodule 1.0 cm or more in largest diameter if microcalcifications are present and (2) a nodule 1.5 cm or more in largest diameter if any of the following apply: (a) nodule is solid or almost entirely solid or (b) there are coarse calcifications within the nodule.

Consider FNA for (1) a nodule 2.0 cm or more in largest diameter if any of the following apply: (a) the nodule is mixed solid and cystic or (b) the nodule is almost entirely cystic with a solid mural component; or (2) the nodule has shown substantial growth since prior US examination. FNA is likely unnecessary if the nodule is almost entirely cystic, in the absence of the above-listed features.
Multiple nodules:
Consider FNA of one or more nodules, with selection prioritised on the basis of the previously stated criteria in the order listed above. FNA is likely unnecessary in diffusely enlarged glands with multiple nodules of similar US appearance without intervening normal parenchyma.
Note that these recommendations are not absolute or inflexible. In certain circumstances, the physician’s clinical judgment may lead him or her to determine that FNA need not be performed for nodules that meet the recommendations above. In others, FNA may be appropriate for nodules that do not meet the criteria listed above.
Part II
The recommendation for non-diagnostic aspirates from initial FNA is as follows: Consider a second FNA of nodules meeting the criteria for FNA of solitary nodules, as outlined above.
Part III
The presence of abnormal lymph nodes overrides the US features criteria and should prompt biopsy of the lymph node and/or (if necessary) of an ipsilateral thyroid nodule.

99
Q
  1. A 62-vear-old man is found to have a right-sided pleural effusion on imaging. Which of the following pathologies is more likely to result in a pleural effusion on the right side than on the left side?
    A. Transection of the proximal thoracic duct
    B. Traumatic rupture of the thoracic aorta (distal to left subclavian artery)
    C. Spontaneous oesophageal rupture
    D. Gastric carcinoma
    E. Pancreatitis
A
  1. A. Transection of the proximal thoracic duct
    Disruption of the thoracic duct results in a chylothorax. The thoracic duct measures 45 cm and commences at the level of T12, ascending on the right side of the aorta, crossing the midline at the level of T5 and ascending in the left hemithorax to drain into the confluence of the left subclavian and internal jugular veins. Accordingly, disruption of the proximal thoracic duct results in a right-sided pleural effusion, and disruption of the distal thoracic duct causes a left-sided pleural effusion. Dissecting aneurysm of the thoracic aorta and traumatic rupture of the aorta both result in left-sided effusions, as does spontaneous oesophageal rupture (Boerhaave syndrome) and gastric carcinoma. Pancreatitis can give rise to bilateral or unilateral pleural effusions, with more than two-thirds of pancreatitis-related effusions occurring on the left.
100
Q
  1. A 26-year-old diabetic man who is known to binge drink is referred to you with deranged liver function tests following antibiotics for a chest infection. On ultrasound, you see a hyperechoic area anteriorly within the left lobe of liver within Segment 4. Which of the MRI sequences from the list below will be the most helpful in determining the nature of this lesion (although all would be required for a definitive diagnosis)?
    A. In-phase and out-of-phase T1-weighted images
    B. T1 and T2W images
    C. Diffusion-weighted images and apparent diffusion coefficient
    D. Dynamic contrast-enhanced phase images
    E. T2 with TE of 80 ms and T2 with TE of 160 ms
A
  1. A. In-phase and out-of-phase T1-weighted images
    The hyperechoic area seen on ultrasound anteriorly within Segment 4 is most commonly caused by focal fatty infiltration and is a common finding at ultrasound and is more common in heavy drinkers, hyperlipidaemia and in obese patients. It is also commonly seen at the portal bifurcation. Although all of the above sequences will be important in accurately defining the lesion, the most useful sequence when this is likely to be focal fatty infiltration will be the in-phase and out-of-phase T1-weighted images. Focal fat should lose signal between the in-phase T1-weighted images and the out-of-phase T1-weighted images.
101
Q
  1. When investigating adrenal disease, which of the following statements is incorrect?
    A. Biochemical evaluation is usually the first line of investigation.
    B. Delayed radiolabelled analogues of cholesterol, such as 1-6β-iodomethyl-19-norcholesterol (NP-59), are used to identify and localise masses that result in adrenal cortical dysfunction.
    323
    324
    C. MIBG, an analogue of guanethidine, is used to image adrenal cortical disorders.
    D. 131I and 123I-MIBG can be used to screen the whole body for sympathomedullary tissue.
    E. 18F fluorodeoxyglucose (FDG) can sometimes differentiate between benign adenomas and metastases.
A
  1. C. MIBG, an analogue of guanethidine, is used to image adrenal cortical disorders.
    All the statements are correct except option C. MIBG, an analogue of guanethidine, is used
    to image adrenal medullary disorders. 18F FDG has been reported to be able to differentiate between benign adenomas and metastases.
102
Q
  1. A 20-year-old male patient presents to the A&E department following a skateboard injury. He has pain in his right clavicle and plain films are subsequently performed, which reveal elevation of the clavicle above the superior border of the acromion. What statement is true in regard to this injury?
    A. The acromioclavicular (AC) ligament is ruptured but the coracoclavicular (CC) ligament is intact.
    B. The joint capsule is ruptured but the CC ligament is intact.
    C. The CC ligament is ruptured but the AC ligament is intact
    D. The AC ligament, CC ligament and joint capsule are ruptured but the deltoid attachment is intact.
    E. The AC ligament and CC ligament are ruptured with detachment of the deltoid muscle.
A
  1. E. The AC ligament and CC ligament are ruptured with detachment of the deltoid muscle. The Rockwood classification is used to classify ACJ injuries. Lifting of the clavicle above the
    superior border of the acromion implies a severe strain injury with typical complete derangement of the internal attachments and stabilising structures (Grade III injury). Grade IV VI injuries also imply the same degree of soft-tissue injuries, but there is accompanying complete dislocation with the clavicle displacing into various positions.
    Grade I injuries have an intact CC with AC ligament strain. Grade II injuries (elevation of the clavicle but not above the superior border of the acromion) have a ruptured AC ligament and joint capsule, with strain of the CC ligament.
103
Q
  1. You are reporting a CT pulmonary angiogram of 39-year-old man who presented with shortness of breath. You have excluded pulmonary embolism but are concerned about the appearances of the trachea, which has diffuse narrowing of the lumen with marked wall thickening. You decided to contact the A&E clinician to clarify patient symptoms. After further questioning, the patient admits to episodes of wheeze and stridor for over past few months. All of the following cause diffuse thickening of the wall with decreased diameter of the tracheal lumen, except:
    A. Relapsing polychondritis
    B. Tracheopathia osteoplastica
    C. Tracheal Wegener’s granulomatosis
    D. Mounier-Kuhn disease
    E. Tracheal sarcoidosis
A
  1. D. Mounier-Kuhn disease
    Tracheobronchomegaly, known as Mounier- Kuhn disease, is a rare condition, usually affecting men in the fourth and fifth decade of life. It is characterised by marked dilatation of the trachea and bronchi. Patients typically present with recurrent respiratory infections and bronchitis. It is caused by atrophy and dysplasia of the tracheal wall and the tracheal diameter is usually more than 3 cm.
    Tracheopathia osteoplastica is a rare benign disease affecting the trachea and characterised by multiple submucosal osteocartilaginous growths within the anterior and lateral walls. This condition, as the others listed above (relapsing polychondritis, tracheal Wegeners granulomatosis, tracheal sarcoidosis), cause significant narrowing of the tracheal diameter with wall thickening and symptoms of stridor, wheeze, cough and dyspnoea
104
Q
  1. A patient presents with deranged liver function tests, right upper quadrant pain, nausea, flushing and recurrent bouts of diarrhoea. The surgeon thinks the patient may have gallstone disease and requests a MRCP. A single 3 mm gallstone is noted within the gallbladder, but the bile ducts are within normal limits. A markedly hyperintense lesion on T2 is noted within the mesentery and a few of the small bowel loops in this region appear tethered. The lesion enhances avidly following administration of gadolinium. There are further hyperintense lesions noted throughout the liver, which also avidly enhance in the arterial phase. Which one of the following is the most likely diagnosis?
    A. Metastatic melanoma
    B. Cholangiocarcinoma with liver and mesenteric metastasis
    C. Retractile mesenteritis
    D. Carcinoid syndrome
    E. Cholecystitis with mesenteric and liver abscesses
A
  1. D. Carcinoid syndrome
    Carcinoid syndrome is a collection of symptoms that are caused by a small proportion of carcinoid tumours. This nearly always occurs following metastasis to the liver, although it can happen if a carcinoid tumour occurs outside the gut and its excreted chemicals (adrenocorticotropic hormone, histamine, serotonin, etc.) bypass liver metabolism. These chemicals cause recurrent diarrhoea, bronchospasm, flushing and right-sided heart failure. A spiculated mesenteric mass with calcification and a desmoplastic effect on the adjacent bowel in a patient with these symptoms is characteristic of carcinoid syndrome. These tumours are often hyperintense on T2-weighted magnetic resonance imaging as they are very’ vascular. They also produce hypervascular metastases. If there is carcinoid syndrome, the carcinoid tumour is usually located in small bowel.
    Malignant melanomas are often bright on T1 and do not usually give a desmoplastic mesenteric reaction or give flushing. Cholangiocarcinomas produce hypervascular liver metastases but mesenteric deposits are unusual and the biliary tree appeared normal in this case. Retractile mesenteritis would not cause liver lesions. Mesenteric abscesses from cholecystitis would be rare and only the rim should enhance with gadolinium. This condition would also not be expected to produce a desmoplastic reaction.
105
Q
  1. An adult patient undergoes ultrasound examination of the abdomen. He is known to have end-stage renal failure and has been on renal dialysis for 5 years. Incidentally, several round and well-defined anechoic lesions arc demonstrated in both kidneys. These demonstrate posterior acoustic enhancement. There is no previous relevant family history and a previous ultrasound 5 years ago was normal. Which of the following is the most likely diagnosis?
    A. Adult polycystic kidney disease
    B. Multicystic dysplastic kidney
    C. Multiple simple renal cysts
    D. Cystic renal cell cancer
    E. Uraemic cystic disease
A
  1. E. Uraemic cystic disease
    About 40% of patients with end-stage renal disease develop renal cysts. The incidence increases with time on dialysis so that incidence of uraemic cystic disease is 90% in patients 5 years on dialysis. Associated complications include haemorrhage within the cysts and rarely malignancy. Cysts regress after successful transplantation.
106
Q
  1. An 80 year old patient with history of rheumatoid arthritis, has been involved in a 40 mph head-on car crash. Air bags were deployed. You are asked to review a cervical spine trauma series. No fracture is identified. The patient has mid-cervical spine tenderness. The patient wants to go home. Which of the following is the best option?
    A. Let the patient be discharged since trauma scries is normal.
    B. Repeat lateral and AP views.
    C. Request a flexion and extension lateral cervical spine.
    I). Cervical spine CT.
    E. Bone scintigraphy
A
  1. D. Cervical spine CT.
    In high-risk patients or patients with limited physical examination, given the relative insensitivity of plain films, CT should be performed. In a rheumatoid patient there is an increased risk of cervical injury. Repeat views would not be of benefit Age and mechanism of injury would indicate CT cervical spine according to NICE guidelines.
107
Q
  1. All of the following statements are true with regard to stress fractures in young athletic children, except
    A. Children with limb misalignment are at greater risk.
    B. Stress fracture of the femoral neck involves the superior surface.
    C. Shin splints show linear oedema limited to the medial tibia.
    D. Distal femoral metaphysis are recognised sites for stress fracture.
    E. Pars interarticularis fractures are due to repetitive extension and torsion.
A
  1. B. Stress fracture of the femoral neck involves the superior surface.
    The most common cause of stress fractures is a chronic and repeated workload. Children with extremity malalignment or abnormal weight-bearing also are at increased risk. Typical locations of stress fractures in children include the tibia, fibula, femur and tarsal and metatarsal bones.
    MR imaging is currently the best diagnostic modality for stress fractures.
    Stress fractures of the femur tend to occur after skeletal maturity and resemble adult injuries, affecting the inferior surface of the neck, the shaft and the distal metaphysis. This injury is most common in endurance athletes, such as runners, triathletes or soccer players but also occurs in association with abnormal weight-bearing, such as with a coxa vara deformity.
    The most common site for stress fractures in the adolescent athlete is the tibia. Tibial stress fractures occur with activities requiring sudden stops or changes in direction, such as football, soccer and tennis.
    Stress also can result in shin splints, which are probably an early stress response secondary to periosteal traction. In cases of stress fractures, MR imaging shows diffuse and irregular bone marrow oedema, whereas in shin splints, the area of high signal intensity often is more linear and is limited to the medial aspect of the tibia.
    Spondylolysis is a stress injury of the pars interarticularis that is due to repetitive extension and torsion of the trunk. Usually occurring in the lower segments of the lumbar spine, stress injuries of the pars interarticularis have been observed in young female gymnasts, college football players and wrestlers.
108
Q
  1. A 7-year-old boy presents to his emergency department complaining of a limp and pain in his left hip. He is febrile and blood test results reveal raised inflammatory markers. The emergency department staff are concerned regarding an underlying diagnosis of septic arthritis.
    Which one of the following statements regarding imaging in septic arthritis is false?
    A. An ultrasonography can be performed to guide needle aspiration of an underlying hip effusion.
    B. Destruction of underlying bone is normally seen at presentation on plain radiography.
    C. The suggestion of underlying hip effusion may be identified on plain radiography of the pelvis.
    D. An effusion normally collects in the anterior recess of the hip joint on ultrasound early in the process.
    E. Septic arthritis is usually a mono-articular process.
A
  1. B. Destruction of underlying bone is normally seen at presentation on plain radiography. If joint inflammation is limited to a single joint, infection must first be carefully excluded. The
    cause of septic arthritis is usually related to haematogenous seeding owing to staphylococcal or streptococcal microorganisms.
    The radiographic features of a septic joint encompass those of any inflammatory arthritis namely, periarticular osteopenia, uniform joint space narrowing, soft-tissue swelling and bone erosions. Not all findings may be present simultaneously, and, acutely, bone erosions may not be
    evident. Furthermore, the joint space may be initially widened owing to the effusion. Joint space widening may also be seen with more indolent and atypical infections, such as those related to tuberculosis and fungal agents.
    The Phemister triad describes these findings classically seen in tuberculous arthritis: juxta- articular osteopenia, peripheral bone erosions and gradual narrowing of the joint space.
    The presence of anterior capsular distension was noted as convexity of the anterior recess and compared to contralateral normal side in equivocal cases.
109
Q
  1. A 42-year-old female patient with shortness of breath undergoes CT pulmonary angiogram for suspected pulmonary embolism (PE). In addition to the presence of subsegmental PE, the scan reveals two well-defined, rounded intraluminal polypoid masses on the posterior wall of the trachea and multiple small nodules in both lungs. Disseminated malignancy is suspected. Which primary malignancy is most likely to metastasize to the trachea?
    A. Malignant melanoma
    B. Ovarian carcinoma
    C. Sarcoma
    D. Transitional cell carcinoma of the urinary bladder
    E. Pancreatic carcinoma
A
  1. A. Malignant melanoma
    About 90% of all tracheal tumours are malignant and of those metastases are the most common group. Malignant melanoma, renal cell tumour, breast carcinoma and colonic adenocarcinoma are the most common sources of haematogenous metastatic spread to the tracheal mucosa. The lesions are usually multiple, polypoid, well defined and without tracheal wall thickening or extratracheal extension, as commonly seen in primary tracheal tumours. Cancers of the thyroid, oesophagus, larynx and lung may invade the tracheal wall by direct extension from the primary source, and this type of spread is the most common in secondary tracheal malignancies. The symptoms of tracheal malignancies are non-specific and include haemoptysis, dyspnoea, chest pain, cough and wheeze.
110
Q
  1. A middle-aged woman presents with a painful anterior neck lump, associated with a rash. Ultrasound of the thyroid demonstrates an enlarged hypoechoic and hypervascular gland. Follow-up imaging 4 months later shows resolution of the previous changes. What is the diagnosis?
    A. Hashimoto’s thyroiditis
    B. De Quervain’s thyroiditis
    C. Multinodular goitre
    D. Papillary thyroid carcinoma
    E. Anaplastic carcinoma
A
  1. A. Hashimoto thyroiditis
    Hashimoto thyroiditis, also known as chronic autoimmune lymphocytic thyroiditis, is a disease with a typical clinical presentation of painless diffuse enlargement of the thyroid gland accompanied by hypothyroidism and thyroid autoantibodies. The sonographic appearance of Hashimoto thyroiditis is well recognised. The gland is often diffusely enlarged, and the parenchyma is coarsened, hypoechoic and often hypervascular. A micronodular pattern on ultrasound is highly diagnostic of Hashimoto thyroiditis with a positive predictive value of 95%. Discrete nodules may, however, also occur within diffusely altered parenchyma or within sonographically normal parenchyma. The nodular form of Hashimoto thyroiditis has not received nearly as much analysis as the diffuse form, and the reported findings have been variable.
111
Q
  1. A 72 year-old woman with B12 deficiency and achlorhydria has a barium meal. The stomach is featureless with no demonstrable rugal folds, and there is narrowing of the body of the stomach. Which of the following is the most likely diagnosis?
    A. Infectious gastritis
    B. Menetrier disease
    C. Eosinophilic gastritis
    D. Atrophic gastritis
    E. Linitis plastica
A
  1. D. Atrophic gastritis
    Atrophic gastritis has a high association with B12 deficiency. It occurs in the older population and is often caused by autoimmune disease. However, it can also be caused by Helicobacter pylori, leading to chronic inflammation of the gastric mucosa and eventually fibrosis. Atrophic gastritis particularly affects the chief and parietal cells, resulting in achlorhydria and B12 deficiency, and causes the classic featureless stomach. It is associated with an increased risk of gastric carcinoma.
    Infectious gastritis, Menetrier disease and eosinophilic gastritis are often associated with thickened folds. Linitis plastica would have a history of weight loss and iron deficiency anaemia as opposed to B12 deficiency.
112
Q
  1. A 26-year-old obese and unwell woman attends the A&E department with right-sided abdominal pain. She has a positive pregnancy test. The A&E doctor requests an abdominal and pelvic ultrasound, which demonstrates some fluid within Morrison’s pouch. The pelvic area is not well demonstrated on transabdominal ultrasound due to the patient’s obesity, but no intrauterine pregnancy is noted. Which of the following should happen next?
    A. Laparotomy
    B. CT of the abdomen and pelvis
    C. Transvaginal ultrasound
    D. MRI of the pelvis
    E. Laparoscopy
A
  1. C. Transvaginal ultrasound
    This patient has a suspected ectopic pregnancy, which is a potentially fatal gynaecological emergency. Transvaginal ultrasound (TVUS) can detect intrauterine pregnancy 1 week sooner than transabdominal ultrasound (TAUS). If no intrauterine pregnancy (IUP) is seen on TAUS, you should therefore confirm that there is no IUP by TVUS where possible. The free fluid in this case is the result of a ruptured bleeding tube secondary to an ectopic pregnancy; the fluid may be echogenic because of blood content. TVUS is preferable to CT, as the patient may have a viable early pregnancy. In hospitals where TVUS is not available, emergency laparoscopy may be considered. However, MRI is not a safe environment for a potentially unstable patient.
113
Q
  1. Which one of the following is not a radiographic hallmark of degenerative arthritis?
    A. Asymmetrical joint space narrowing
    B. Subchondral sclerosis
    C. Periarticular osteoporosis
    D. Subchondral cysts
    E. Osteophytes
A
  1. C. Periarticular osteoporosis
    Degenerative arthritis and osteoarthritis are interchangeable terms that can be used to describe the clinical and radiographic joint changes associated with ageing.
    Early changes include damage to the articular cartilage with progressive damage to and loss of the ground substance that eventually leads to the subchondral bone being exposed
    Clinically, patients present with joint pain worsened with use and the joint may be found on examination to be enlarged, deformed, tender with crepitus and possibly display joint effusion.
    The five radiographic hallmarks are asymmetrical joint space narrowing, subchondral sclerosis, subchondral cysts, osteophytes and lack of osteoporosis. Periarticular osteoporosis is much more characteristic of rheumatoid arthritis.
114
Q
  1. A 30-year-old teacher presents with thyroid swelling and proptosis. Which of the following is true regarding Grave’s ophthalmopathy?
    A. It commonly involves the medial rectus first.
    B. It commonly involves the tendons of the eye muscles.
    C. There is associated dilatation of the superior ophthalmic vein.
    D. There is decreased density of the orbital fat.
    E. It is an autoimmune disease unrelated to thyroid function.
A
  1. C. There is associated dilatation of the superior ophthalmic vein.
    Graves’ disease of the orbit is also known as thyroid ophthalmopathy or endocrine exophthalmos. It is produced by long-acting thyroid stimulating hormone, probably due to cross reactivity against antigens shared by thyroid and orbital tissue. Signs and symptoms usually develop within 1 year of hyperparathyroidism, it is the most common cause of unilateral/bilateral proptosis in adults. It commonly involves the inferior rectus first (mnemonic I’M SLOW). The superior ophthalmic vein is dilated due to compromised orbital venous drainage at the orbital apex.
    There is increased density of the orbital fat late in the disease.
115
Q
  1. A 78-year-old man presents with weight loss and frank haematuria. He had a stricture of his urethra dilated many years ago. He has no other relevant past history. Initial ultrasound and IVU show a normal upper urinary tract and bladder. As part of the standard work-up, he proceeds to flexible cystoscopy, but the urologist is unable to pass the cystoscope beyond the proximal penile urethra, lie is therefore referred for an ascending urethrogram, which demonstrates an irregular filling defect in the bulbar urethra. Which of the following is the likeliest diagnosis?
    A. Prostatic carcinoma
    B. Transitional cell carcinoma of the urethra
    C. Squamous cell carcinoma of the urethra
    D. Adenocarcinoma of the urethra
    E. Recurrent benign stricture
A
  1. C. Squamous cell carcinoma of the urethra
    Primary urethral carcinoma is rare. About 50%-75% occur in the bulbar urethra and nearly 90% are squamous type, particularly if there has been a stricture in the past. Urethral carcinoma is often advanced at diagnosis, explaining the weight loss. Primary transitional cell carcinomas (TCC) are closer to the bladder neck, although secondary TCCs can occur anywhere in the urethra when seeded from the bladder in patients with a bladder TCC history, when they are often multiple (these are the most commonly encountered male urethral tumours). Adenocarcinomas are extremely rare and tend to occur at the prostatic utricle or glands of Littre. Haematogenous metastases to the urethra are exceedingly rare but are most likely to be from melanoma. Prostatic carcinoma would be more likely to involve the prostatic urethra. Benign strictures can appear a little irregular, but weight loss would not be expected.
116
Q
  1. A 34-year-old patient with a recent history of a prolonged stay in an intensive care unit following road traffic accident is referred to the respiratory clinic with symptoms of dyspnoea, expiratory wheeze and recurrent chest infections. A CT scan of the chest is performed.
    Which one of the following is the most likely tracheal complication of the previous prolonged intubation in this patient?
    A. Carrot-shaped trachea
    B. Sabre-sheath trachea
    C. Tracheomalacia
    D. Tracheopathia osteoplastica
    E. Tracheal diverticulum
A
  1. C. Tracheomalacia
    Weakening of the tracheal walls associated with posterior tracheal wall collapse during expiration is a recognised complication post-intubation and tracheal tube placement, when the
    cuff pressures exceed 25 cmH20. This condition is often underdiagnosed as it can present with non-specific symptoms such as shortness of breath, wheeze, stridor and recurrent chest infections. Dynamic CT scan during inspirational and expirational phases is the modality of choice for its diagnosis, and it is considered positive when the AP diameter of the trachea decreases more than 50% during expiration. Tracheomalacia may also be a complication of chronic obstructive pulmonary disease (COPD), relapsing polychondritis and recurrent infections.
    Tracheal diverticulum and saber-sheath trachea (decreased coronal diameter of trachea) are conditions associated with COPD, and carrot-shaped trachea refers to a narrowing of the tracheal diameter in the caudal direction as a result of an aberrant left pulmonary artery.
117
Q
  1. You are performing a small bowel follow-through on a 42-year-old female patient because of abdominal pain, weight loss and anaemia. The patient has had a normal endoscopy and colonoscopy. There is separation of the ileal loops within the right side of the abdomen. Some of the ileal loops in this region appear compressed on one side and one of the loops appears aneurysmal. You also note the presence of a fistula between the distal ileum and colon. Except for these ileal loops, the remainder of the small bowel appears normal. Which one of the following is the most likely diagnosis?
    A. Crohn’s disease
    B. Tuberculosis
    C. Small bowel lymphoma
    D. Metastatic serosal deposits
    E. Coeliac disease
A
  1. C. Small bowel lymphoma
    Small bowel lymphoma is a soft pliable tumour and therefore can often be quite large in size without causing a bowel obstruction. Characteristically, it destroys Auerbach’s plexus, leading to aneurysmal dilatation of the small bowel; however, this is a rare finding. It often fistulates, mimicking Crohn’s disease, but there would be more likely to be skip lesions with Crohn’s disease. Tuberculosis can have similar appearance, but these patients will often be from an endemic area or have HIV; fever and night sweats will also be more prominent, although these symptoms can also occur in lymphoma.
118
Q
  1. A 65-year-old man presents with pain, swelling and paraesthesia in his left thigh. Clinical examination revealed a large 10 x 6 cm soft-tissue mass within the left thigh. The mass is of soft-tissue density on CT and demonstrates avid contrast enhancement, ’there is no involvement of the adjacent bone. On MRI, the mass returns a predominantly low signal on T1W sequences and high signal on T2W sequences. Which of the following is the most likely diagnosis?
    A. Lipoma
    B. Liposarcoma
    C. Osteosarcoma
    D. Soft-tissue fibroma
    E. Lipoblastoma
A
  1. B. Liposarcoma
    Liposarcoma is a malignant tumour of mesenchymal origin; it is the second most common soft-tissue sarcoma in adults after malignant fibrous histiocytoma (now’ called pleomorphic undifferentiated sarcoma or PUS). It usually presents in the fifth to sixth decade. On CT, it is seen as a non-specific soft-tissue mass, as the fat content is usually not radiologically detectable. On MRI, because of the high content of myxoid cells (the most common type of liposarcoma), it is low signal on T1-weighted sequences and high signal on T2-weighted sequences.
    A lipoma would not be expected to enhance post-contrast. A large lymph node would be unlikely to reach the sizes described and additionally may be hyperintense to muscle on T1-weighted sequences. A soft-tissue fibroma would have a small hypointense nodule on all sequences. Lipoblastoma usually affects children less than 3 years of age.
119
Q

@#e 119. On a routine neonatal birth check, the paediatric registrar suspects an infant of having developmental dysplasia of the hip (DDH), and refers the him for a hip ultrasound.
Which one of the following statements regarding hip ultrasound according to the Graf classification is correct?
A. A normal alpha angle is less than or equal to 60 degrees.
B. A normal beta angle is greater than 20 degrees.
C. The alpha angle is measured from the triangular labral fibrocartilage to the vertical cortex of the ilium.
D. The beta angle can help to determine the dysplasia subtype.
E. The acetabular angle is measured from the acetabular roof to the vertical cortex of the ilium.

A
  1. D. The beta angle can help to determine the dysplasia subtype.
    The Graf method for ultrasound classification system for developmental dysplasia of the hip (DDH) in infants combines both the alpha and beta angles. The alpha angle is formed by the acetabular roof to the vertical cortex of the ilium.
    The normal value is greater than or equal to 60 degrees. Less than 60 degrees suggests acetabular dysplasia.
    The beta angle is defined as the angle formed between the vertical cortex of the ilium and the triangular labral fibrocartilage (echogenic triangle). There is a great deal of variability in the beta angle and is not universally used.

The beta angle is necessary to determine the subtype of hip dysplasia.
Acetabular angle is a radiographic assessment for dysplasia. It is formed by a horizontal line connecting both triradiate cartilages (Hilgenreiner line) and a second line which extends along the acetabular roofs.

120
Q
  1. CT of the head of a young man for chronic headache showed erosion of the left frontal sinus. Post-contrast showed focal meningeal enhancement with mucosal enhancement of the right frontal sinus. What is the most likely diagnosis?
    A. Mucocoele
    B. Inverted papilloma
    C. Pott’s puffy tumour
    D. Antrochoanal polyp
    E. HIV
A
  1. C. Pott’s puffy tumour
    Pott’s puffy tumour is defined as a subperiosteal abscess of the frontal bone with frontal osteomyelitis.
    Frontal sinus infection can spread directly through the thin bone wall of this sinus or through the network of small veins that drain its mucosa. Today, this is a rare complication, given the widespread use of antibiotics. Trauma and frontal sinusitis arc the most common causes of this condition.
    The bacteria causing Pott’s puffy tumour usually reflect the type of bacterial species responsible for community-acquired chronic sinusitis.
    The most common causal organisms are streptococci, staphylococci and anaerobic bacteria. Cultures frequently reveal polymicrobial involvement. The infection may spread as a thrombophlebitis from the frontal sinus through the diploic veins, involving the intracranial space with consequent epidural or subdural empyema, meningitis, brain abscess and venous sinus thrombosis.